You are on page 1of 45

Contact us : info@onlyias.

com
Visit : dpp.onlyias.in
Contact : +91-7007 931 912

DEDICATED PRELIM PROGRAM(DPP)


UPSC-CSE PRELIMINARY TEST SERIES – 2021/2022
DO NOT OPEN THIS TEST BOOKLET UNTIL YOU ARE ASKED TO DO SO
TEST BOOKLET SERIES

Test Code:

E5
TEST BOOKLET

DPP GENERAL STUDIES


TEST -05

Time Allowed: Two Hours Maximum Marks : 200

INSTRUCTIONS
1. IMMEDIATELY AFTER THE COMMENCEMENT OF THE EXAMINATION, YOU SHOULD CHECK THAT THIS TEST
BOOKLET DOES NOT HAVE ANY UNPRINTED OR TORN OR MISSING PAGES OR ITEMS, ETC.
2. ENCODE CLEARLY THE TEST BOOKLET SERIES CODE A, B, C OR D AS THE CASE MAY BE IN THE
APPROPRIATE PLACE IN THE ANSWER SHEET.
3. You have to enter your Roll Number on the Test Booklet in the Box
provided alongside this line. DO NOT write anything else on the
Test Booklet.
4. This Test Booklet contains 100 questions. Each question is printed in English. Each item comprises four
responses (answers). You will select the response which you want to mark on the Answer Sheet. In case you feel
that there is more than one correct response, mark the response which you consider the best. In any case, choose
ONLY ONE response for each item.
5. All items carry equal marks.
6. Penalty for wrong answers:
THERE WILL BE PENALTY FOR WRONG ANSWERS MARKED BYA CANDIDATE IN THE OBJECTIVE TYPE
QUESTION PAPERS.
I. There are four alternatives for the answer to every question. For each question for which a wrong
answer has been given by the candidate, One-Third of the marks assigned to that question will be
deducted as penalty.
II. If a candidate gives more than one answer, it will be treated as a wrong answer even if one of the given
answers happens to be correct and there will be same penalty as above to that question.
III. If a question is left blank, i.e., no answer is given by the candidate, there will be no penalty for that
question.

DO NOT OPEN THIS TEST BOOKLET UNTIL YOU ARE ASKED TO DO SO


TOPICS COVERD: Election Commission of India Comptroller and Auditor General (CAG),Attorney General of
India Advocate General of the State, Finance Commission National Commission for Backward
Classes,National Commission for Scheduled Castes National Commission for Scheduled Tribes,
Special officer for Linguistic Minorities Union Public Service Commission State Public Service
Contact us : info@onlyias.com

OnlyIAS Nothing Else Visit : dpp.onlyias.in


Contact : +91-7007 931 912

Q.1) With reference to the National commission for 1. Manner of removal of the chief election
backward classes, consider the following statements: commissioner.
1. It is a non-constitutional body. 2. Qualification of the members of the election
2. It is a statutory body under the Ministry of Social commission
Justice & Empowerment. 3. Total Term of the members of the election
3. The conditions of service and tenure of office of commission
the Chairperson, Vice-Chairperson and other 4. Various conditions of service of the chief election
Members is determined by the President. commissioner
Select the correct Ans: using the codes given below:
Which of the above statements is/are correct? a) 1 only
b) 1 and 2 only
a) 1 only c) 1 and 3 only
b) 3 only d) 2 and 4 only
c) 1 and 3 only
d) 1, 2 and 3 Q.5) With reference to Election Commission of India,
consider the following statements:
Q.2) Consider the following options: 1. The constitution provides security of tenure to all
1. Elections to the Lok Sabha Constituency of South the members of the Election Commission of India.
Mumbai 2. The constitution puts an obligation on the
2. Elections to the sole Rajya Sabha seat of governor of a state to make available to the
Puducherry election commission such personnel as may be
3. Elections to the Legislative Assembly of Delhi necessary for various functions.
4. Greater Hyderabad Municipal Corporation 3. The constitution of India has debarred the retiring
Elections election commissioners from any further
5. Uttar Pradesh Gram Panchayat Raj Election appointment under the government.
6. Elections to the Legislative Council of Karnataka Which of the above statements is/ are correct?
Which of the above elections are conducted by the a) 1 and 2 only
Election Commission of India? b) 2 and 3 only
a) 1,2 and 3 only c) 1 and 3 only
b) 3,4,5 and 6 only d) 2 only
c) 1,2,3 and 6 only
d) All of the above Q.6) With reference to the National Commission on
backward class which of the following statements
Q.3) Which of the following are functions of the is/are incorrect?
election commission of India?
1. Determine the territorial areas of the electoral 1. The NCBC is a statutory body established under
constituencies the Ministry of Social Justice and Empowerment.
2. Grant recognition to political parties 2. The first Backward classes commission or the
3. Deregister political parties for unconstitutional Mandal commission recommended that the
practices members of OBC be granted reservations to 27 %
4. Cancel polls in the event of rigging booth capturing of jobs under the Central government and public
and violence sector undertakings.
Select the correct Ans: using the codes given below: 3. Kalekar commission or the socially and
a) 1, 2 and 3 only Educationally Backward Classes was established
b) 1, 3 and 4 only with a mandate to “identify the socially or
c) 2, 3 and 4 only educationally backward classes” of India.
d) 1, 2 and 4 only
Select the Answer using the codes given below:
Q.4) With reference to Election Commission of India, a) 1 and 2 only
which of the following are explicitly mentioned in the b) 2 and 3 only
constitution? c) 1,2 and 3 only

DAY 34_IDMP FULL LENGTH TEST 5 1


Contact us : info@onlyias.com

OnlyIAS Nothing Else Visit : dpp.onlyias.in


Contact : +91-7007 931 912

d) None of the above major policy matters affecting the socially and
educationally backward classes.
Q.7) Consider the following statements about 2. The President can specify the socially and
National Commission for Backward Classes (NCBC): educationally backward classes with respect to any
1. It is a Constitutional body established originally in State.
2018 under Article 338B of constitution. 3. The union government, by notification can include or
2. It consists of a chairperson and 3 other members. exclude any socially and educationally backward class
3. It investigates and monitors all the matters relating from the Central list of socially and educationally
to economically backward classes under the backward classes.
Constitution. Which of the above statement(s) is/are correct?
Which of the above statements are incorrect? a) 1 only
a) 1 and 2 only b) 1 and 2 only
b) 1 and 3 only c) 2 and 3 only
c) 2 and 3 only d) None of the above
d) 1, 2 and 3
Q.11) Consider the following statements:
Q.8) Consider the following statements about 1. The members of the National Commission for
National Commission for Backward Classes: Backward Classes (NCBC) have collective responsibility.
1. It has all the powers of a civil court trying a suit. 2. The Secretary of the NCBC, in his discretion, can
2. The Commission can regulate its own procedure. delegate any of his functions or authority to a
3. The chairperson of the Commission has the residuary subordinate officer without the approval of the
powers to decide on all questions and matters arising Chairperson.
in the Commission. 3. NCBC cannot take action in the cases in which a
Which of the above statement(s) is/are correct? Court has already given its final verdict.
a) 1 and 2 only Which of the above statement(s) is/are correct?
b) 1 and 3 only a) 1 only
c) 2 only b) 1 and 3 only
d) 1, 2 and 3 c) 2 and 3 only
d) 3 only
Q.9) Consider the following statements about the
functions of National Commission for Backward Q.12) Consider the following statements about
Classes: National Commission for Scheduled Castes (NCSC):
1. It can participate and advise on the planning process 1. It has been given constitutional status by 88th
of socio-economic development of the socially and Constitutional Amendment Act.
educationally backward classes. 2. It consists of a chairperson and three other
2. It can evaluate the progress of development of members.
socially and educationally backward classes under the 3. NCSC has to investigate all matters relating to the
Union but not under the States. constitutional and other legal safeguards for the Anglo-
3. It presents its reports pertaining to union Indian community.
government to the President and those pertaining to Which of the above statement(s) is/are correct?
state government to the Governor. a) 1 and 3 only
Which of the above statement(s) is/are correct? b) 1 and 2 only
a) 1 and 3 only c) 3 only
b) 1 only d) 1 only
c) 1, 2 and 3 only
d) None of the above Q.13) Consider the following statements about
National Commission for Scheduled Castes:
Q.10) Consider the following provisions made by the 1. It can participate and advise on the planning process
102nd Constitution Amendment Act, 2018: of socio-economic development of the scheduled
1. The Union and State governments need to consult castes.
the National Commission for Backward Classes on all

DAY 34_IDMP FULL LENGTH TEST 5 2


Contact us : info@onlyias.com

OnlyIAS Nothing Else Visit : dpp.onlyias.in


Contact : +91-7007 931 912

2. It can evaluate the progress of development of c) 1 and 3 only


scheduled castes under the Union but not the States. d) 1, 2 and 3
3. It has all the powers of a civil court and can summon
a person from any part of India. Q.17) Consider the following statements about the
Which of the above statement(s) is/are correct? National Commission for Scheduled Tribes (NCST):
a) 1 and 3 only 1. The Commission takes measures to reduce and
b) 1 only eliminate the practice of shifting cultivation by tribals.
c) 1, 2 and 3 only 2. The Commission may hold more than one sitting
d) None of the above simultaneously in different parts of the country.
3. Any matter on which there is no provision in the
Q.14) Consider the following provisions about rules of procedure, the decision of the President shall
scheduled castes under the Constitution: be sought.
1. The Union and State governments need to consult Which of the above statements are correct?
the National Commission for Scheduled Castes on all a) 1 and 2 only
major policy matters affecting the scheduled castes. b) 2 and 3 only
2. The President can specify the scheduled castes with c) 1 and 3 only
respect to any State. d) 1, 2 and 3
3. The President, by law may include or exclude any
caste, race or tribe from the list of scheduled caste Q.18) Consider the following statements about the
specified in notification. provisions made by the Constitution with reference to
Which of the above statement(s) is/are correct? Union Public Service Commission (UPSC):
a) 1 only 1. The UPSC consists of a chairperson and nine other
b) 1 and 2 only members.
c) 2 and 3 only 2. The chairperson should be a person who has held
d) 1, 2 and 3 office for at least ten years under the Government of
Q.15) Consider the following statements about India.
National Commission for Scheduled Castes (NCSC): 3. The chairperson and members of the UPSC hold
1. The Minister of Social Justice and Empowerment is office for a term of six years and until they attain the
the chairperson of the NCSC. age of 65 years.
2. The Commission cannot take up investigations Which of the above statements is/are correct?
directly and has to conduct them through state a) 1 and 2 only
agencies. b) 3 only
3. The Commission has to meet at least once in a c) 1 and 3 only
month. d) 1, 2 and 3
Which of the above statement(s) is/are correct?
a) 1 and 2 only Q.19) In which of the following instances the Union
b) 2 and 3 only Public Service Commission may not be consulted?
c) 3 only 1. While making reservations of appointments or posts
d) 1, 2 and 3 in favour of any backward class of citizens.
2. While taking into considerations the claims of the
Q.16) Consider the following statements about scheduled castes and scheduled tribes in making
National Commission for Scheduled Castes: appointments to services and posts.
1. The Commission cannot take up matters that are 3. All disciplinary matters affecting a person serving
related to transfer. under the Government of India.
2. The Complaint for violation of reservation policy can 4. Any claim for award of a pension in respect of
be directly addressed to the chairman or vice chairman injuries sustained by a person while serving under the
of commission. Government of India.
3. The Commission can regulate its own procedure. Select the correct Ans: using the code given below:
Which of the above statements are correct? a) 1 and 2 only
a) 1 and 2 only b) 2 and 3 only
b) 2 and 3 only c) 1 and 4 only

DAY 34_IDMP FULL LENGTH TEST 5 3


Contact us : info@onlyias.com

OnlyIAS Nothing Else Visit : dpp.onlyias.in


Contact : +91-7007 931 912

d) 1, 2 and 3 only c) 2 and 3 only


d) 1, 2 and 3
Q.20) Consider the following statements about State
Public Service Commission (SPSC) and Joint State Q.23) Consider the following statements about the
Public Service Commission (JSPSC): removal of chairperson or any member of a State
1. SPSC is a constitutional body whereas JSPSC is a Public Service Commission:
statutory body. 1. They can be removed by the President in a like
2. The upper age limit to hold the office for the manner of a High Court Judge.
members of SPSC is 62 years, whereas it is 65 years for 2. ‘Misbehaviour’ is one of the grounds for removal and
the members of JSPSC. is defined in the Constitution.
3. The chairperson and members of both SPSC and 3. During the course of enquiry by the Supreme Court,
JSPSC can be removed by the President only. the Governor can suspend the concerned chairperson
4. SPSC presents the report on annual performance to or member.
the Governor of the state and JSPSC presents the Which of the above statements are correct?
report on annual performance to the President. a) 1 and 2 only
Which of the above statements is/are correct? b) 2 and 3 only
a) 1, 3 and 4 only c) 1 and 3 only
b) 1 and 3 only d) 1, 2 and 3
c) 2 and 4 only
d) 1, 2 and 4 only Q.24) Consider the following statements about the
Special Officer for Linguistic Minorities:
Q.21) Consider the following statements about State 1. The provision for the Special Officer for Linguistic
Public Service Commission: Minorities has been made under Part XVI (Special
1. The Constitution does not specify the strength of the provisions relating to certain classes) of the
Commission but has left the matter to the discretion of Constitution.
the President. 2. The Special Officer is appointed by the President of
2. The Constitution authorises the President to India.
determine the conditions of service of the chairperson 3. The Constitution provides that the Special Officer
and members of the Commission. reports to the President annually and at such intervals
3. The Governor can appoint one of the members of as the President may direct.
the SPSC as an acting chairperson in case of vacancy Which of the above statement(s) is/are correct?
and absence of the chairperson. a) 1 and 3 only
Which of the above statement(s) is/are incorrect? b) 2 only
a) 1 only c) 2 and 3 only
b) 3 only d) 1, 2 and 3
c) 1 and 2 only
d) 1 and 3 only Q.25) Consider the following statements about the
Q.22) Consider the following statements about State Special Officer for Linguistic Minorities:
Public Service Commission: 1. The provision for the Special Officer for Linguistic
1. An Act of State Legislature can provide for exercise Minorities was made through the 7th Constitutional
of additional functions as respects the services of any Amendment Act.
local authority. 2. The Constitution provides the Special Officer should
2. The SPSC is consulted by the Governor while framing be a person from the group of linguistic minorities and
rules for appointment to judicial service of the state having 10 years of experience in the Government of
other than the posts of district judges. India or a state.
3. The chairperson of the SPSC, on ceasing to hold 3. The Special Officer can investigate a matter when
office, is eligible for appointment as the chairperson or children belonging to a linguistic minority group are not
a member of the UPSC. provided instruction in the mother-tongue at the
Which of the above statement(s) is/are correct? primary stage of education.
a) 2 only Which of the above statements are correct?
b) 1 and 3 only a) 1 and 2 only

DAY 34_IDMP FULL LENGTH TEST 5 4


Contact us : info@onlyias.com

OnlyIAS Nothing Else Visit : dpp.onlyias.in


Contact : +91-7007 931 912

b) 2 and 3 only Select the correct Ans: using the codes below.
c) 1 and 3 only a) 1, 2 and 3 only
d) 1, 2 and 3 b) 1 and 4 only
c) 1, 3 and 4 only
Q.26) The prior recommendation of the President is d) 2, 3 and 4 only
needed to introduce which of these bills in the
Parliament? Q.29) Consider the following statements regarding
1. A bill involving expenditure from the Consolidated Attorney General of India
Fund of India. 1. Its office is mentioned in the Article 148 of the
2. A bill for the alteration of boundaries of states. Constitution.
3. A private member bill that concerns minority 2. He/she receives the Remuneration as fixed by the
welfare. Parliament.
4. Any ordinary bill in which the states may be 3. He/she is under the pleasure of the President.
interested. Which of the statements given above is/are correct?
a) 1 and 2
Select the correct Ans: using the codes below. b) 3 only
c) 2 and 3
a) 1 and 2 only d) 1 and 3
b) 3 and 4 only
c) 1, 2 and 3 only Q.30) Consider the following statements regarding
d) 1, 2 and 4 only Attorney General of India
1. The terms of office of the AG is not fixed by the
Constitution.
Q.27) Article 123 of the Constitution empowers the 2. Constitution does not provide the grounds for
President to promulgate ordinances. Consider the his/her removal.
following in this regard. 3. He/she should be a retired chief justice of India.
1. An ordinance made when both the Houses are in Which of the statements given above is/are correct?
session is valid if approved by the Council of a) 1 and 3
Ministers and Parliament. b) 1 and 2
2. An ordinance cannot be used to amend the c) 2 only
constitution. d) 1, 2 and 3
Which of the above is/are correct?
Q.31) Consider the following statements
a) 1 only
1. Attorney General does not fall in the category of
b) 2 only
government servants
c) Both 1 and 2
2. He can defend accused persons in criminal
d) None of the above
prosecutions without the permission of the
Government of India.
3. He enjoys all the immunities and privileges that are
Q.28) The Governor has constitutional discretion in
available to a member of Parliament
which of the Following cases?
Which of the statements given above is/are correct?
1. Seeking information from the chief minister with
a) 1 only
regard to the administrative and legislative
b) 1 and 3
matters of the state
c) 2 and 3
2. Approving any ordinary bill that falls under the
d) 1, 2 and 3
concurrent list
3. Recommendation for the imposition of the
Q.32) Consider the following statements regarding
President’s Rule in the state
Advocate General of State
4. Exercising his functions as the administrator of an
1. He must be a person who is qualified to be
adjoining union territory in case of additional
appointed a judge of Supreme Court.
charge
2. He holds office during the pleasure of Governor.

DAY 34_IDMP FULL LENGTH TEST 5 5


Contact us : info@onlyias.com

OnlyIAS Nothing Else Visit : dpp.onlyias.in


Contact : +91-7007 931 912

3. He has a fixed tenure of 3 years as prescribed by Q.36) With reference to the Welfare of the Backward
the constitution. Classes (OBCs) in India, consider the following
Which of the statements given above is/are incorrect? statements:
a) 1 only 1. It is one of the target groups of the Ministry of
b) 1 and 3 Social Justice and Empowerment.
c) 2 and 3 2. The Constitution of India does not provide a
d) 3 only specific definition of what constitutes the
‘Backward Classes’(BCs)
Q.33) With reference to National Commission of 3. Article 338B of the Constitution of India provides a
Backward classes, consider the following statements: national Commission for the welfare of the
1. It is a Constitutional body established through ‘Backward Classes’(BCs)
the One Hundred and first constitutional Which of the statements given above is/are correct?
Amendment. a) 1 only
2. The pay and rank of members of the Commission b) 1 and 3 only
are those of Secretary to the Govt of India. c) 2 and 3 only
Which of the statements given above is/are correct? d) 1, 2 and 3
a) 1 only
b) 2 only Q.37) With reference to the National Commission for
c) Both 1 and 2 Scheduled Castes, consider the following statements:
d) Neither 1 nor 2 1. The conditions of service of office of the members
of the Commission are determined by the
Q.34) With reference to the composition of the President.
National Commission of Backward Classes, consider 2. It is empowered to enforce the attendance of any
the following statements: person from any part of India while inquiring into
1. The rules provide for the mandatory appointment any complaint.
of at least one-woman member in the 3. It is a multi-member body directly established by
Commission. the Constitution of India.
2. The Chairperson of the commission is appointed Which of the statements given above is/are correct?
from amongst eminent socio-political workers a) 1 and 2 only
belonging to the socially and educationally b) 2 and 3 only
backward classes. c) 1 and 3 only
3. The Members of the Commission are not eligible d) 1, 2 and 3
for appointment for more than one term.
Which of the statements given above is/are correct? Q.38) With reference to the National Commission for
a) 1 only Scheduled Castes, which one of the following
b) 1 and 2 only statements is incorrect?
c) 2 and 3 only a) The separate National Commission for Scheduled
d) 1, 2 and 3 Castes (SCs) came into existence as per the 65th
Constitutional Amendment Act of 1990.
Q.35) With reference to the National Commission of b) The Commission submits its annual report to the
the Backward Classes which one of the following President of India.
statements is incorrect? c) The state governments are required to consult the
a) The Chairperson and the Members of the Commission on all major policy matters affecting
Commission are appointed by the President by the SCs.
warrant under his hand and seal. d) The Commission has to investigate matters related
b) It has all the powers of a civil court trying a suit. to legal safeguard of Anglo-Indian Communities.
c) The Commission is mandated by the Constitution
to specify the socially and educationally backward Q.39) With reference to the National Commission for
classes. Scheduled Tribes, consider the following statements:
d) None of the Above

DAY 34_IDMP FULL LENGTH TEST 5 6


Contact us : info@onlyias.com

OnlyIAS Nothing Else Visit : dpp.onlyias.in


Contact : +91-7007 931 912

1. The term of office of Chairperson and each Q.43) Consider the following statements with respect
member is five years from the date of assumption to Election Commission
of charge. 1. The Constitution itself has provided power to the
2. The Chairperson of the Commission has been given Election commission to demand necessary staff
the rank of Union Cabinet Minister. from the President and Governor.
Which of the statements given above is/are correct? 2. Election commission has power to take disciplinary
a) 1 only action against the officers for insubordination or
b) 2 only dereliction of duty while on election duty
c) Both 1 and 2 3. Election commission has been given power to
d) Neither 1 nor 2 superintendence, direction and control of the
preparation of electoral rolls for all elections to
Q.40) With reference to the National Commission for state legislature
Scheduled Tribes, consider the following statements: Which of the above statements is/are correct?
1. It is a five-member body with a Chairperson, Vice- a) 1 and 2 only
Chairperson and three other Members b) 2 and 3 only
2. The Commission is vested with powers of a civil c) 1 and 3 only
court. d) None of the above
3. The Union Minister of Tribal Affairs serves as an Ex-
officio Vice-Chairperson of the Commission. Q.44) With reference to Article 324, consider the
Which of the statements given above is/are correct? following statements
a) 1 and 2 only 1. Power conferred under Article 324 is subject to
b) 2 only limitations When Parliament or any State
c) 1 and 3 only Legislature has made a valid law relating to or in
d) 1, 2 and 3 connection with elections
2. While exercising power, Election commission shall
Q.41) Consider the following statement regarding be confirmed to the rule of law and amenable to
Election commission of India norms of natural Justice
1. Constitution at its enactment provided for a multi Which of the above statement(s) is/are incorrect?
member body of Election commission of India. a) 1 only
2. Election commission has essentially Administrative b) 2 only
and marginally Legislature as well as Judicative c) Both 1 & 2
powers. d) Neither 1 nor 2
Which of the above statements is/are Correct?
a) 1 only Q.45) Consider the following statements:
b) 2 only 1. Word elections in Article 324 includes every
c) Both 1 & 2 process issued after the issuance of the
d) Neither 1 nor 2 Notification for holding the election.
2. Election commission can withhold election to state
Q.42) Consider the following statements legislature if state government failed to complete
1. Free and fair elections is one of the basic structures the process of issuance of photo identity cards by
propounded by the judiciary. the deadline.
2. The two Election commissioner has equal status to 3. Constitution has empowered the Election
that of Chief Election commissioner. commissioners to delete the name of a person
3. Tenure of Both Election commissioner and regional from the Electoral Roll-on certain grounds.
commissioner is free of political Executive control. Which of the above statements is/are Incorrect?
Which of the statement/s given above is/are correct? a) 1 only
a) 1 only b) 1 and 2 only
b) 1 and 3 only c) 2 and 3 only
c) 2 only d) 1, 2 and 3
d) 1, 2 and 3

DAY 34_IDMP FULL LENGTH TEST 5 7


Contact us : info@onlyias.com

OnlyIAS Nothing Else Visit : dpp.onlyias.in


Contact : +91-7007 931 912

Q.46) With reference to Finance Commission, 1. Recommendations of Finance commission are


consider the following statements implemented both by Presidential order as well as
1. The Finance Commission's work involves Execution order
equalization of all public services across the States. 2. Recommendations related to grant in aid and
2. Finance Commission's work involves redressing the distribution of Union Taxes and Duties has to be
vertical imbalances between the taxation powers implemented by an executive order.
and expenditure responsibilities of the centre and Which of the above statement(s) is/are correct?
the States respectively.
Which of the statement/s given above is/are Correct? a) 1 only
a) 1 only b) 2 only
b) 2 only c) Both 1 & 2
c) Both 1 and 2 d) Neither 1 nor 2
d) Neither 1 nor 2
Q.50) Consider the following statements about
Q.47) Which of the following, is duty of Finance finance commission:
commission to make recommendations to the 1. Financial or other interest which is likely to affect
President prejudicially to a person's functions as a member is
1. Distribution of net proceed of taxes between a ground for disqualification of member of Finance
Centre and State. commission
2. Principles governing grant in aid of revenue of 2. President shall satisfy himself that person will have
states out of consolidated fund of States. no such financial or other interest as is likely to
3. Measures needed to augment consolidated fund of affect prejudicially his functions as a member
state to supplement resources of municipalities on Which of the above statement(s) is/are incorrect?
the basis of recommendation of state Finance a) 1 only
commission. b) 2 only
4. any other matter referred to the Commission by c) Both 1 & 2
the President in the interests of sound finance. d) Neither 1 nor 2
Choose the correct Ans: from the code given below:
a) 1, 2, 3 and 4 Q.51) With reference to Finance Commission,
b) 1, 3 and 4 only consider the following statements
c) 2 and 4 only 1. Commission, in the performance of their function,
d) 1 and 3 only has the power of Civil court under Codes of Civil
procedure
Q.48) With reference to Finance commission, consider 2. Commission can summon and enforce the
the following statements attendance of witnesses
1. Chairman of Finance Commission is selected as per 3. Commission has power to require any person to
qualification mentioned in Article 280 of furnish information on such points or matters as in
constitution the opinion of the Commission may be useful or
2. Special knowledge of finance and accounts of relevant to any matter under its consideration
government or qualification to be Judge of HC are Which of the above statement(s) is/are correct?
one of the prescribed conditions of selection for a) 1 only
members of the finance commission. b) 2 only
Which of the above statement(s) is/are correct? c) 1 and 3 only
d) 1, 2 and 3 only
a) 1 only
b) 2 only Q.52) With reference to Union Public Service
c) Both 1 & 2 Commission, consider the following statements
d) Neither 1 nor 2 1. Misbehaviour is the only ground for removal of
member or chairperson of Union Public Service
Q.49) Consider the following statements Commission.

DAY 34_IDMP FULL LENGTH TEST 5 8


Contact us : info@onlyias.com

OnlyIAS Nothing Else Visit : dpp.onlyias.in


Contact : +91-7007 931 912

2. If a member or chairman becomes in any way 2. UPSC shall be consulted, on any claim for the
concerned or interested in any contract or award of a pension in respect of injuries sustained
agreement made by or on behalf of the by a person while serving under the Government
Government of India in any way in profit, he would of India
be deemed guilty for misbehaviour. Which of the above statement(s) is/are correct?
Which of the above statement(s) is/are correct? a) 1 only
a) 1 only b) 2 only
b) 2 only c) Both 1 and 2
c) Both 1 & 2 d) Neither 1 nor 2
d) Neither 1 nor 2
Q.56) With reference history of the office of
Q.53) With reference to Union Public Service Comptroller and Auditor general of India (CAG),
Commission, consider the following statement consider the following statements
1. Parliament may regulate the number of members 1. Govt of India Act 1858 record the first reference to
of commission and their condition of service. the office of Auditor General of India
2. Conditions of service of a member of a Public 2. Montford Reform 1918 made the Office of CAG
Service Commission cannot be varied to his independent from the sovereign
disadvantage after his appointment. 3. Govt of India order 1936 laid down condition of
3. Expenses of Union public service commission is service of Auditor general and defined his duties
charged on consolidated fund of India. and powers
Which of the above statement(s) is/are correct? Which of the above statement(s) is/are correct?
a) 1 only a) 1 and 2 only
b) 2 only b) 2 and 3 only
c) 2 and 3 only c) 1 and 3 only
d) 1 and 3 only d) 1, 2 and 3

Q.54) Consider the following statements regarding Q.57) With reference to the office of CAG, consider
public service commissions the following statements
1. On ceasing to hold office Chairman of a State public 1. In 1976, office of CAG was made responsible for
service commission (SPSC) can become member of accounting and auditing of both the Central and
Union public service commission (UPSC) the State governments in pursuance of the quasi-
2. On ceasing to hold office Member of State Public federal nature of the country
Service Commission can become chairperson of 2. Salary of CAG finds mention in second schedule to
Union public service commission (UPSC) the Constitution of India
3. On ceasing to hold office Member of UPSC can Which of the above statement(s) is/are correct?
become member of SPSC a) 1 only
4. On ceasing to hold office Chairman of SPSC can b) 2 only
become chairman of another SPSC c) Both 1 & 2
Which of the above statement(s) is/are correct? d) Neither 1 nor 2

a) 1, 2 and 3 only Q.58) Consider the following statements regarding


b) 1, 3 and 4 only office of CAG
c) 1, 2 and 4 only 1. CAG is appointed by warrant under hand and seal
d) 2, 3 and 4 only of President.
2. CAG of India doesn't take oath.
Q.55) With reference to public service commissions, 3. It is the President who prescribe the form in which
consider the following statements the accounts of Union and states shall be
1. State legislature may provide for exercise of maintained and kept.
additional function related to services of state-by- Which of the above statement(s) is/are correct?
State public service commission a) 1 and 3 only
b) 2 and 3 only

DAY 34_IDMP FULL LENGTH TEST 5 9


Contact us : info@onlyias.com

OnlyIAS Nothing Else Visit : dpp.onlyias.in


Contact : +91-7007 931 912

c) 1 only 3. Ministry of social justice and Empowerment


d) 1, 2 and 3 only coordinates all activities related to schedule caste
and scheduled tribe.
Which of the above statement(s) is/are correct?
Q.59) Consider the following statement with
reference to the power of CAG a) 1 and 3 only
1. CAG is authorised to audit accounts related to b) 2 only
expenditure from consolidated funds of India and c) 3 only
Consolidated funds of all States. d) 2 and 3 only
2. CAG can also audit public account of India but not
Contingency fund of India Q.63) With reference to Complaint to National
3. CAG can also audit local bodies on request of the commission for scheduled Tribes, Consider the
president or governor. following statements
Which of the statement/s given above is/are correct? 1. Complainant should disclose his full Identity and
a) 1 only give his full address while filing a complaint
b) 1 and 3 only 2. Complains can't be directly addressed to the
c) 3 only chairman or vice chairman of the commission
d) 1, 2 and 3 3. Complains for which there is case pending in courts
need to be taken afresh with the commission
Q.60) Consider the following statements Which of the above statement(s) is/are correct?
1. CAG derives all its powers from constitution. a) 1 and 2 only
2. Duties and powers conferred by the Constitution b) 2 only
on the CAG under Article 149 is basic structure of c) 1 only
constitution as propounded by the judiciary. d) None of the above
Which of the statement/s given above is/are
incorrect? Q.64) Which of the following functions are not
a) 1 only performed by National commission for Scheduled
b) 2 only Tribe?
c) Both 1 & 2 a) To investigate and monitor matters relating to the
d) Neither 1 nor 2 constitutional and other legal safeguards for the
STs.
Q.61) With reference to National commission for b) Inquire specific complaints relating to deprivation
Scheduled Caste, consider the following statements of rights of ST’s.
1. Year 1978 saw modification of function and c) Declare Schedule tribe in an area in consultation
rename of the of National commission for SCs & with the governor.
STs (scheduled Tribe) d) To present to the President, annually and at such
2. A separate National Commission for SCs came into other times as it may deem fit, reports upon the
existence in 2004. working of those safeguards.
Which of the statement/s given above is/are correct?
a) 1 only Q.65) Which of the following doesn't come under the
b) 2 only ambit of the power of National commission for
c) Both 1 & 2 scheduled Caste as a civil court
d) Neither 1 nor 2 a) Summon any person from any part of the country
b) Requiring the discovery and production of any
Q.62) Consider the following statements document;
1. In 1960, Lokur committee laid down standards to c) Receiving evidence on affidavits;
put any community or caste in tribal group. d) None of the above
2. According to 2011 census, Scheduled Tribe account
for about 8.6% of country's population. Q.66) Consider the following statements with
reference to National commission for Backward
classes:

DAY 34_IDMP FULL LENGTH TEST 5 10


Contact us : info@onlyias.com

OnlyIAS Nothing Else Visit : dpp.onlyias.in


Contact : +91-7007 931 912

1. Article 338 - B provides for National commission 1. Original constitution didn't Make any provision
for Backward Classes for socially and economically with regard to special officer for linguistic
backward classes. minorities
2. Before it's establishment, power to look into 2. 7th Constitutional amendment gave way for it
complaints and welfare measures with regard to through insertion of Article 350-B
OBC was entrusted to National commission for 3. Constitution has not specified any of tenure, salary,
scheduled Tribe. allowance, service conditions and removal
3. Kalelkar Commission (1953) was the first to procedure for the special officer for linguistic
identify backward classes other than the minorities
Scheduled Castes and Scheduled Tribes at the Which of the above statement/s is/are correct:
national level. a) 1 only
Which of the above statement(s) is/are incorrect? b) 2 and 3 only
a) 1 and 2 only c) 1 and 2 only
b) 2 and 3 only d) 1, 2 and 3
c) 1 and 3 only
d) None of above
Q.70) Consider the following statements with
Q.67) With reference to Attorney General, consider reference to Comptroller and Auditor general of India
the following statements (CAG)
1. His consent is required to initiate proceedings for 1. Though appointment by President, the service
criminal contempt of Supreme Court in case conditions are determined by the Parliament.
motion is brought by a person other than himself. 2. A cabinet minister can represent CAG in
2. His consent is mandatory when a private citizen parliament.
wants to initiate a case of contempt of court 3. Only proved Misbehaviour or Incapacity can be the
against a person. ground for removal of CAG.
3. The Supreme Court cannot take suo motu Which of the statement given above is/are correct?
cognizance in case of criminal contempt of a) 1 and 2 only
Supreme Court. b) 2 and 3 only
Which of the above statement(s) is/are correct? c) 1 and 3 only
a) 1 and 3 only d) 1, 2 and 3
b) 2 and 3 only
c) 1 and 2 only Q.71) In the context of the Uniform Civil Code,
d) None of the above consider the following statements:

Q.68) Consider the following statements with 1. It is the exclusive right of the Parliament to enact
reference to Attorney General of India laws governing personal laws of the citizens.
1. Constitution neither has provided a fix term of
2. Article 25 of the Constitution protects the rights of
tenure nor prescribed the procedure for removal.
the individuals and not that of the religious
2. He has right of audience in proceedings of
denominations.
parliament without right to vote.
3. He can't defend an accused person in any civil or
3. The S. R. Bommai case separated religion from the
criminal proceedings.
secular functions of the states.
Which of the above statement(s) is/are correct?
a) 1 and 2 only Which of the statements given above is/are correct?
b) 1 and 3 only
c) 2 and 3 only a) 1 and 2 only
d) 1, 2 and 3 only b) 2 and 3 only
c) 1 and 3 only
Q.69) Consider the following statements with d) 1, 2, and 3
reference to Special Officer for linguistic minorities
Q.72) Consider the following statements:

DAY 34_IDMP FULL LENGTH TEST 5 11


Contact us : info@onlyias.com

OnlyIAS Nothing Else Visit : dpp.onlyias.in


Contact : +91-7007 931 912

1. Both Parliament and State legislatures can enact a Q.75) In relation to the Unlawful Activities
law under Article 16(3) to permit domicile based (Prevention) Act, 1967, which of the statements given
preferential treatment. below is/are incorrect?

2. Nagraj case (2006) judgement of the Supreme court 1. Investigation of cases under this Act is the sole
called for the quantifiable data on the backwardness of responsibility of the National Investigation Agency.
the SC/ST.
2. It empowers both Centre and State governments to
Which of the statements given above is/are incorrect? impose a ban on organizations.

a) 1 only 3. Both Individuals and organizations can be


b) 2 only designated as terrorists under this Act.
c) Both 1 and 2
d) Neither 1 nor 2 Select the correct Ans: using the codes given below:

Q.73) Consider the following pairs: a) 1 and 2 only


b) 2 and 3 only
Species IUCN Conservation c) 1 and 3 only
Status d) 1, 2, and 3

1. Forest Elephant Vulnerable Q.76) Consider the following pairs:

2. Asian Elephant Endangered Country Ports

3. Dugong Vulnerable 1. Seychelles Anjouan

4. Olive Ridley Turtles Critically Endangered 2. Madagascar Ehoala

Which of the pairs given above is/are correctly 3. Bangladesh Mongla Port
matched?
Which of the pairs given above is/are correct?
a) 3 only
b) 2 and 3 only a) 1 and 2 only
c) 3 and 4 only b) 2 and 3 only
d) 1, 2 and 4 only c) 1 and 3 only
d) 1, 2, and 3
Q.74) Consider the following statements:
Q.77) Consider the following statements regarding
1. PM Swasthya Suraksha Nidhi seeks to provide a Quasi-Resonant Amplification (QRA):
source of funding for the Ayushmann Bharat
Programme. 1. It is caused due to warming around the temperate
and tropical latitudes.
2. The proceeds of the PMSSN forms a part of the
Consolidated Fund of India. 2. There has been no evidence of Heat waves
associated with QRA from India.
Which of the statements given above is/are correct?
Which of the statements given above is/are correct?
a) 1 only
b) 2 only a) 1 only
c) Both 1 and 2 b) 2 only
d) Neither 1 nor 2 c) Both 1 and 2
d) Neither 1 nor 2

DAY 34_IDMP FULL LENGTH TEST 5 12


Contact us : info@onlyias.com

OnlyIAS Nothing Else Visit : dpp.onlyias.in


Contact : +91-7007 931 912

Q.78) Consider the following: a) 1 and 3 only


b) 3 only
1. Easy rechargeability c) 1 and 2 only
2. Lower cost d) 2 and 3 only
3. Capable of covering longer distances.
4. Recyclability Q.82) In the context of Quadrilateral Security
5. Less energy density Dialogue, consider the following statements:

Which of the above factors are favorable for the 1. It is a military grouping of the countries having
adoption of Aluminum Air batteries? converging interests in the Indo-Pacific region.
2. It came into existence on the sidelines of the East
a) 1, 2, 3, and 5 Asian Summit of 2017 in the Philippines.
b) 1, 3, and 5
c) 2, 3, and 4 Which of the statements given above is/are correct?
d) 1, 2, 3, and 4
a) 1 only
Q.79) Consider the following statements regarding b) 2 only
Millets in India: c) Both 1 and 2
d) Neither 1 nor 2
1. India is the second largest producer of Millets after
China. https://www.financialexpress.com/defence/quad-
2. Millets are usually grown in areas of higher rainfall summit-2021-why-is-china-rattled/2212544/
above 500 mm.
3. Rajasthan accounts for the largest area under Millet Q.83) Consider the following pairs:
cultivation in India.
1. Red Sea Shatt Al- Arab
Which of the statements given above is/are incorrect?
2. Persian Gulf Sharm El Sheikh
a) 1 only
b) 2 and 3 only 3. Mediterranean Sea Port Said
c) 1 and 2 only
d) 2 only 4. Gulf of Oman Muscat

Q.80) Dhaudhan Dam, Orr Dam recently in news are Which of the pairs given above is/are correctly
related to which of the following? matched?

a) Tapi- Narmada River linking a) 1, 3, and 4 only


b) Mahanadi- Godavari River linking b) 2, 3, and 4 only
c) Kosi-Ghagra River linking c) 3 and 4 only
d) Ken-Betwa River linking d) 2 and 3 only

Q.81) In the context of the Indus Water Treaty, Q.84) Consider the following statements:
consider the following statements:
1. The addition of South Africa to BRICS occurred post
1. Tarbela and Mangla Dam projects were envisaged the formation of the New Development Bank.
in this treaty.
2. The Gulf Cooperation Council was formed by all
2. It provides rights to India to develop run off the
bordering countries of the Persian Gulf.
river projects on the western rivers.
3. Indus and Beas are located at the Northern and Which of the statements given above is/are correct?
Southern ends of the Indus River Systems.
a) 1 only
Which of the statements given above is/are correct?
b) 2 only

DAY 34_IDMP FULL LENGTH TEST 5 13


Contact us : info@onlyias.com

OnlyIAS Nothing Else Visit : dpp.onlyias.in


Contact : +91-7007 931 912

c) Both 1 and 2 3. Regular increase in the prices of the stocks held by


d) Neither 1 nor 2 a person attracts Capital Gains Tax.

Q.85) The terms Apatani, Zabo, Dighis, Paar, Jampois Which of the statements given above is/are correct?
are related to?
a) 1 and 3 only
a) Traditional system of agriculture and irrigation. b) 2 and 3 only
b) Traditional water harvesting systems of India. c) 2 only
c) Tribes associated with the Island territories of India. d) 1 and 2 only
d) Tribal groups of the Northeastern states of India.
Q.89) Consider the following statements:
Q.86) Consider the following statements:
1. Electric Vehicles equipped with batteries are
1. Commercial Banks and Development Finance exempted from the levy of GST to enable their easy
Institutions are incorporated under the Companies adoption.
Act. 2. The Vehicle scrapping policy deals with both the
2. National Bank for Financing Infrastructure and commercial and private vehicles only.
Development aims at generating funds in foreign
currencies only. Which of the statements given above is/are incorrect?
3. It finances the long-term infrastructure projects
situated wholly within the geographical borders of the a) 1 only
country. b) 2 only
c) Both 1 and 2
Which of the statements given above is/are correct? d) Neither 1 nor 2

a) 1 only Q.90) Which of the statements given below is most


b) 2 and 3 only appropriate regarding the National Food Security Act,
c) 3 only 2013?
d) None of the above
a) There has been a continuous updation of the
Q.87) Consider the following statements: Central Issue Price (CIP) of the food grains after every
three years.
1. National Payment Corporation of India has been b) It is the responsibility of the Central government to
incorporated as the Critical Payment Infrastructure issue licenses to the fair price shops for the purpose
Company of India. of the distribution.
2. Cyber Swachhta Kendra has been launched to c) Shanta Kumar Committee suggested a revision in
detect and remove the malicious software and other the list of beneficiaries to include a greater number of
related threats. persons.
d) A higher food subsidy bill is attributed to the cost of
Which of the statements given above is/are correct? procurement and distribution being greater than the
CIP.
a) 1 only
b) 2 only Q.91) Consider the following statements:
c) Both 1 and 2
d) Neither 1 nor 2 1. As per the Global Hunger Index (GHI), almost 50% of
Indian children suffer from stunting.
Q.88) Consider the following statements:
2. Bangladesh has been accorded a better rank than
1. Greater Bond Yields on the government bonds India in both the GHI and the World Happiness Index.
drive up the prices of the stocks at the share market.
2. A higher bond yield of the USA government might Which of the statements given above is/are correct?
reduce investments in other economies.

DAY 34_IDMP FULL LENGTH TEST 5 14


Contact us : info@onlyias.com

OnlyIAS Nothing Else Visit : dpp.onlyias.in


Contact : +91-7007 931 912

a) 1 only a) 1 only
b) 2 only
c) Both 1 and 2 b) 2 only
d) Neither 1 nor 2 c) Both 1 and 2
d) Neither 1 nor 2
Q.92) Consider the following statements regarding
the Large Hadron Collider: Q.95) Consider the following statements:

1. Large Hadron Collider deals with the study of the 1. Supercomputers exhibit parallel processing of
fundamental particles and the detection of Black information unlike the ordinary computers that
Holes. exhibit serial processing.
2. Particles at the Large Hadron Collider are made to 2. The computing speed of supercomputers is
accelerate at a speed much greater than the speed of measured in FLOPS while that of ordinary computers
light. is measured in bytes.
3. India has been accorded an Observer status at the 3. The speed of the Fugaku supercomputer is approx.
European Organization for Nuclear Research (CERN). ten times more than India’s PARAM Siddhi.

Which of the statements given above is/are incorrect? Which of the statements given above is/are incorrect?

a) 1 and 2 only a) 1 and 2 only


b) 2 and 3 only b) 2 and 3 only
c) 1 and 3 only c) 1 and 3 only
d) 1, 2, and 3 d) 1, 2, and 3

Q.93) Consider the following pairs: Q.96) The Ministry of AYUSH is presently
implementing the Centrally Sponsored Scheme of
Personality Associated with National AYUSH Mission (NAM) which, among other
objectives, aims to support herbal industry bodies
1. Prof. Velcheru Narayan Rao Telugu literature and promote medicinal plants. In this context, The
Govt. of India provides support to farmers in which of
2. King Bhumibol Adulyade Chakri Dynasty the following ways?
1. Funds and machinery for primary processing and
3. Ayya Vaikunda Swamikal Samathwa Samajam
marketing infrastructure.
2. Cultivation of all types of medicinal plants on
4. Lachit Borphukan Ahom Dynasty
farmer’s land.
Which of the pairs given above is/are correctly 3. Establishment of nurseries with backward linkages.
matched? 4. Post-harvest management with forward linkages.
Select the correct Ans: using the code given below:
a) 1, 2, and 3 only a) 1 only
b) 2, 3, and 4 only b) 1,3 and 4 only
c) 1, 2, and 4 only c) 2,3 and 4 only
d) 1, 2, 3, and 4 d) 1,2,3 and 4

Q.94) Consider the following statements: Q.97) Consider the following statements:
1. The production of Raw silk in India has steadily
1. Coronal Mass Ejections are one of the reasons for increased in the last five years.
the failure of satellites in space. 2. The Central Silk Board (CSB) is a statutory body
2. ARKTIKA-M is a Russian endeavor to study the under the Department of Animal Husbandry and
warming of the two polar regions. Dairying.
Which of the above statements is/are correct?
Which of the statements given above is/are correct? a) 1 only
b) 2 only

DAY 34_IDMP FULL LENGTH TEST 5 15


Contact us : info@onlyias.com

OnlyIAS Nothing Else Visit : dpp.onlyias.in


Contact : +91-7007 931 912

c) Both 1 and 2
d) None of the above

Q.98) The Tropic of Cancer passes through which of


the following Harappan Sites?
a) Lothal
b) Dholavira
c) Kalibangan
d) Chanhudaro

Q.99) The Organization of the Petroleum Exporting


Countries (OPEC) and its allies, a group known as
OPEC+ were recently seen in news. Which of the Q.1) Ans: B
following countries belong to the OPEC+ group? Exp:
1. Russia
2. Azerbaijan Statement 1 is incorrect: It is a constitutional body.
3. Yemen 102nd Constitution Amendment Act, 2018 provides
4. China constitutional status to the National Commission for
5. Oman Backward Classes (NCBC).For this purpose amendment
Select the correct Ans: using the code given below: inserted a new article 338-B in the constitution.
a) 1,2 and 3 only
b) 1,2 and 5 only Statements 2 is incorrect: Previously NCBC was a
c) 3,4 and 5 only statutory body under the Ministry of Social Justice and
d) 1,2,3,4 and 5 Empowerment.

Q.100) The Global Climate Risk Index is published by Statement 3 is correct: The Commission consists of five
a) World Economic Forum members including a Chairperson, Vice-Chairperson
b) German watch and three other Members appointed by the President
c) Climate Action network by warrant under his hand and seal. The conditions of
d) United Nations Environment Programme service and tenure of office of the Chairperson, Vice-
Chairperson and other Members is determined by the
President.

Q.2) Ans: C
Exp:
• Options 1, 2, 3 and 6 are correct: The Election
Commission administers elections to the Lok Sabha,
Rajya Sabha, State Legislative Assemblies
(including Delhi), State Legislative Councils and the
offices of the President and Vice President of the
country.
• Option 4 and 5 are not correct: According to Article
243K and Article 243-ZA ,the superintendence,
direction and control of the preparation of electoral
rolls for, and the conduct of, all elections to the
Municipalities and Panchayats shall be vested in
the State Election Commission consisting of a State
Election Commissioner to be appointed by the
Governor

DAY 34_IDMP FULL LENGTH TEST 5 16


Contact us : info@onlyias.com

OnlyIAS Nothing Else Visit : dpp.onlyias.in


Contact : +91-7007 931 912

Q.3) Ans: D Q.4) Ans: A


Exp: Exp:
• Statements 1,2 and 4 are correct: The following is • Statement 1 is correct: According to article 324 of
a non-exhaustive list of the various functions of the the constitution the chief election commissioner
election commission of India: shall not be removed from his office except in like
o To determine the territorial areas of manner and on the grounds as a judge of the
the electoral constituencies supreme court and the conditions of service of the
throughout the country on the basis of chief election commissioner shall not be varied to
the Delimitation Commission act of his disadvantage after his appointment.
parliament. • Statements 2 is incorrect: The constitution has not
o To notify the dates and schedules of prescribed the qualifications ( legal, educational
elections and to scrutinize nomination administrative or judicial ) of the members of the
papers election commission. The supreme court, recently
o To register political parties for the pointed out that, there is a “gap” due to the lack of
purpose of elections and grant them a parliamentary law which transparently spells
the status of national or state parties out the eligibility criteria required for a person to
on the basis of their poll performance. be an election commissioner and that there is a
o To advise the president whether “legitimate expectation in the Constitution of
elections can be held in a state under India” that Election Commissioner should be
president’s rule in order to extend the appointed through the “most transparent and just
period of emergency after one year process” formalised by a law enacted by the
o To advise the president on matters Parliament.
relating to the disqualifications of the • Statements 3 and 4 are incorrect: The constitution
members of parliament. has not prescribed the term and conditions of
o To appoint officers for inquiring into service of the Election Commission members. The
disputes relating to electoral Election commission (CONDITIONS OF SERVICE OF
arrangements. ELECTION COMMISSIONERS AND TRANSACTION
o To act as a court for settling disputes OF BUSINESS)] Act, 1991, stipulates that the
related to granting of recognition to members of the election commission shall hold
political parties and allotment of office for a term of 6 years or until they attend the
election symbols to them age of 65 years, whichever is earlier. The act
o To cancel polls in the event of rigging, further stipulates various conditions of service of
booths capturing, violence and other the members of the election commission.
irregularities.
SOURCE: Bare act : election commission act, 1991

• Statement 3 is incorrect: The ECI in a recent


affidavit, pointed out to the Supreme Court
that Section 29A of the Representation of Q.5) Ans: D
People Act provides for procedure of Exp:
registration of political parties, but it does not • Statement 1 is incorrect: Article 324 provides the
expressly confer any power on the Election Chief Election Commissioner (CEC) only with
Commission regarding deregistration of security of tenure. He cannot be removed from his
political parties. The Election Commission office except in the same manner and on the same
went on to demand that it must be ground as a judge of the supreme court and the
empowered to deregister a political party if it service conditions of the CEC cannot be varied to
violates provisions of the Constitution and his disadvantage after his appointment. The Other
refuses to bring reforms to maintain inner election commissioners however, can be removed
party democracy. from office on the recommendation of the CEC.

DAY 34_IDMP FULL LENGTH TEST 5 17


Contact us : info@onlyias.com

OnlyIAS Nothing Else Visit : dpp.onlyias.in


Contact : +91-7007 931 912

• Statement 2 is correct: According to article 324(6), backward classes. 102nd Constitutional


the Governor of a State, shall, when so requested Amendment Act, 2018 conferred a constitutional
by the Election Commission, make available to the status to the Commission under Article 338B of the
Election Commission or to a Regional Constitution.
Commissioner such staff as may be necessary for • Statement 2 is incorrect: The Commission consists
the discharge of the functions conferred on the of a chairperson, a vice-chairperson and three
Election Commission by clause other members.
• Statement 3 is incorrect: The constitution of India • Statement 3 is incorrect: It investigate and
has not debarred the retiring election monitor all matters relating to the constitutional
commissioners from any further appointment by and other legal safeguards for the socially and
the government. This makes them susceptible to educationally backward classes
political pressure from the ruling government
during their respective terms. Reference: Laxmikanth

Q.6) Ans: C Q.8) Ans: D


Exp: Exp:
• Statement 1 is correct: Article 338B(8) provides all
Statement 1 is incorrect: Initially NCBC was a the powers of a civil court trying a suit to the
statutory body established under the Ministry of Commission.
Social Justice and Empowerment, but later 102nd • Statement 2 is correct: Article 338B(4) provides
Constitution Amendment Act, 2018 provided that the Commission shall have the powers to
constitutional status to the National Commission for regulate its own procedure.
Backward Classes (NCBC). Thus NCBC is a • Statement 3 is correct: The Chairperson is the
Constitutional body. head of the Commission and has the residuary
powers to decide on all questions and matters
Statement 2 is incorrect: The Second Backward arising in the Commission.
classes’ commission or Mandal commission
recommended that the members of OBC be granted Reference: 102nd Constitution Amendment Act, 2018
reservations to 27 % of jobs under the Central http://www.ncbc.nic.in/User_Panel/UserView.aspx?T
government and public sector undertakings. ypeID=1219 – (Rule 2.1.1)
The First backward classes commission is called the
Kaka Kelkar commission.
Statement 3 is incorrect: Mandal commission or the Q.9) Ans: B
Socially and Educationally Backward Classes Exp:
commission was established with a mandate to • Statement 1 is correct: Article 338B (5) (c) assigns
“identify the socially or educationally backward the Commission the duty to participate and advise
classes” of India. on the socio-economic development of the socially
Source: M.LaxmiKanth and Google. and educationally backward classes.
• Statement 2 is incorrect: Article 338B (5) (c)
further provides that the Commission has to
evaluate the progress of development of socially
Q.7) and educationally backward classes under the
Ans: D Union and any State.
Exp: • Statement 3 is incorrect: Article 338B (5) (d)
• Statement 1 is incorrect: The National Commission assigns the Commission the duty to present to the
for Backward Classes was established in 1993, President, annually and at such other times as the
when the Supreme Court in the Mandal Case Commission may deem fit, reports upon the
(1992) directed the Central Government to working of the safeguards for socially and
constitute permanent statutory body to examine educationally backward classes. The President
the complaints of under-inclusion, overinclusion or forwards any report of the Commission pertaining
non-inclusion of any class of citizens in the list of
DAY 34_IDMP FULL LENGTH TEST 5 18
Contact us : info@onlyias.com

OnlyIAS Nothing Else Visit : dpp.onlyias.in


Contact : +91-7007 931 912

to a state government to the state government.


The government places it before the state Q.12) Ans: C
legislature. Exp:
• Statement 1 is incorrect: 65th Constitutional
Reference: Amendment Act, 1990 provided for establishment
102nd Constitution Amendment Act, 2018 of a high-level multi-member National Commission
for SCs and STs. The 89th Constitutional
Q.10) Ans: B Amendment Act, 2003 bifurcated the National
Exp: Commission for SCs and STs into NCSC and NCST.
• Statement 1 is correct: Article 338B (9), which was • Statement 2 is incorrect: NCSC consists of a
added by 102nd Constitution Amendment Act, of chairperson, a vice-chairperson and three other
the Constitution provides that “the Union and members.
State Government shall consult the Commission on • Statement 3 is correct: According to Article 338
all major policy matters affecting the socially and (10) of the Constitution, the Commission has to
educationally backward classes”. investigate all matters relating to the
• Statement 2 is correct: Article 342A (1), which was constitutional and other legal safeguards for the
added by 102nd Constitution Amendment Act, Anglo-Indian Community and report to the
provides that “The President may with respect to President upon their working.
any State or Union territory, and where it is a State,
after consultation with the Governor thereof, by Reference: Laxmikanth
public notification, specify the socially and
educationally backward classes which shall for the
purposes of this Constitution be deemed to be Q.13) Ans: A
socially and educationally backward classes in Exp:
relation to that State or Union territory”. • Statement 1 is correct: Article 338 assigns the
• Statement 3 is incorrect: Article 342A (2), which Commission the duty to participate and advise on
was added by 102nd Constitution Amendment Act, the socio-economic development of the scheduled
provides that Parliament may by law include in or castes.
exclude from the Central List of socially and • Statement 2 is incorrect: Article 338 further
educationally backward classes specified in a provides that the Commission has to evaluate the
notification. progress of development of scheduled castes
Reference: 102nd Constitution Amendment Act, 2018 under the Union and any State.
• Statement 3 is correct: Article 338 provides that
Q.11) Ans: B the Commission has all the powers of a civil court
Exp: trying a suit and can summon and enforce the
• Statement 1 is correct: The Members of the attendance of any person from any part of India
Commission have collective responsibility and and examine him on oath.
function by participating in the 'meetings' and
'sittings' of the Commission and looking after the Reference: Laxmikanth
subjects allocated to them.
• Statement 2 is incorrect: The Secretary of the Q.14) Ans: B
NCBC may, in his discretion, delegate any of his Exp:
functions or authority to a subordinate officer of • Statement 1 is correct: Article 338 (9) of the
the Secretariat with the approval of the Constitution provides that “the Union and State
Chairperson. Government shall consult the Commission on all
• Statement 3 is Correct: The Commission is major policy matters affecting the scheduled
required to inquire into specific complaints with castes”.
respect to the deprivation of rights and violation of • Statement 2 is correct: Article 341(1) provides that
safeguards of socially & educationally backward “The President may with respect to any State or
classes. No action will be taken in the cases in Union territory, and where it is a State, after
which a Court has already given its final verdict. consultation with the Governor thereof, by public

DAY 34_IDMP FULL LENGTH TEST 5 19


Contact us : info@onlyias.com

OnlyIAS Nothing Else Visit : dpp.onlyias.in


Contact : +91-7007 931 912

notification, specify the castes, races or tribes or • Statement 1 is incorrect: The cases of
parts of or groups within castes, races or tribes administrative nature like
which shall be for the purposes of this Constitution transfer/posting/grading of Annual Confidential
be deemed to be scheduled castes in relation to Reports cannot be taken up by the Commission
that State or Union territory”. unless there is caste-based harassment of the
• Statement 3 is incorrect: As per Article 341(2) The petitioner.
Parliament (not president), by law may include or • Statement 2 is correct: As per the rule of
exclude any caste, race or tribe or groups within commission any Complaint within jurisdiction of
castes, races or tribes from the list of scheduled commission should be directly addressed to the
caste specified in notification. chairman/vice chairman/secretary of NCSC, Delhi
or head of its state offices.
• Statement 3 is correct: The commission is vested
Reference: Constitution of India with the power to regulate its own procedure.

Q.15) Ans: C Reference:


Exp: Laxmikanth
• Statement 1 is incorrect: The Minister of Social http://ncsc.nic.in/pages/display/126-rules-of-
Justice and Empowerment is not the chairperson procedure (Rule 7.4.1)
of the NCSC. It has its separate chairperson and
vice-chairperson and three members. They are
appointed by the President by warrant under his Reference: Laxmikanth
hand and seal.
• Statement 2 is incorrect: The Commission may Q.17) Ans: A
adopt any one or more of the following methods Exp:
for investigating or inquiring into the matters • Statement 1 is correct: The Commission has to
falling within its authority: take measures to reduce and ultimately eliminate
o by the Commission directly; the practice of shifting cultivation by tribal that
o by an Investigating Team lead to their continuous disempowerment and
constituted at the degradation of land and the environment.
Headquarters of the • Statement 2 is correct: The Commission may hold
Commission; more than one sitting simultaneously in different
o through its State offices parts of the country with different Members
o by the State Agencies functioning separately.
o by any other • Statement 3 is incorrect: If a question arises
institution/department regarding any such matter for which no provision
funded by the Central exists in the rules of procedure, the decision of the
Government and its statutory Chairperson shall be sought. The Chairperson may,
bodies. if he deems fit, direct that the matter may be
• Statement 3 is correct: The commission has to considered at a meeting of the Commission.
meet at least once in a month. The notice for a
meeting shall normally be issued two weeks in Reference:
advance. Laxmikanth
https://ncst.nic.in/content/rules-
Reference: procedure#:~:text=Clause%204%20of%20the%20Artic
Laxmikanth le,notified%20on%2017%20September%2C%202004.
http://ncsc.nic.in/pages/display/126-rules-of- (Rule 86, 54)
procedure (Rule 7.2, 8.1)

Q.18) Ans: B
Q.16) Ans: B Exp:
Exp:

DAY 34_IDMP FULL LENGTH TEST 5 20


Contact us : info@onlyias.com

OnlyIAS Nothing Else Visit : dpp.onlyias.in


Contact : +91-7007 931 912

• Statement 1 is incorrect: The Constitution does Act of the Parliament on the request of the state
not specify the strength of the UPSC and has left legislature concerned.
the matter to the discretion of the President, who • Statement 2 is incorrect: The chairperson and
determines its composition. Usually, the members of SPSC and JSPSC hold office for a term
commission consists of nine to eleven members of six years or until they attain the age of 62 years.
including the chairman. • Statement 3 is correct: Although the chairperson
• Statement 2 is incorrect: The Constitution does and members of a SPSC are appointed by the
not provide the qualification for the membership Governor, they can be removed only by the
of the commission, except that one-half of the President. The chairperson and members of JSPSC
members of the Commission should be such are appointed and removed by the President.
persons who have held office for at least ten years • Statement 4 is incorrect: The report of SPSC is
either under the Government of India or under the presented to the Governor of the state and the
government of a state. report of JSPSC is presented to each of the
• Statement 3 is correct: Article 316(2) provides that concerned State Governors.
the chairperson and members of the Commission
hold office for a term of six years or until they Reference:
attain the age of 65 years. Laxmikanth

Reference: Laxmikanth Q.21)


Ans: C
Exp:
Q.19) Ans: A • Statement 1 is incorrect: The Constitution does
Exp: not specify the strength of the Commission but has
• Statement 1 is correct: Article 320(4) provides that left the matter to the discretion of the Governor of
the UPSC may not be consulted as respects the the State.
manner in which any provision related to Article • Statement 2 is incorrect: The Constitution
16(4), i.e. reservations of appointments or posts in authorises the Governor to determine the
favour of any backward class of citizens. conditions of service of the chairperson and
• Statement 2 is correct: Article 320(4) further members of the Commission.
provides that the UPSC may not be consulted as • Statement 3 is correct: The Governor can appoint
respects the manner in which effect may be given one of the members of the SPSC as an acting
to the provisions of Article 335, i.e. the claims of chairperson in case of vacancy and absence of the
the scheduled castes and scheduled tribes in chairperson. This provision was added through the
making appointments to services and posts. 15th Constitution Amendment.
• Statement 3 is incorrect: Article 320(3)(c) provides
that the UPSC shall be consulted for all disciplinary Reference: Laxmikanth
matters affecting a person serving under the
Government of India. Q.22) Ans: D
• Statement 4 is incorrect: Article 320(3)(e) provides Exp:
that the UPSC shall be consulted on Any claim for • Statement 1 is correct: Article 321 provides that
award of a pension in respect of injuries sustained the State Legislature may provide for exercise of
by a person while serving under the Government additional functions by the SPSC as respects to the
of India. services of the State and also as respects the
services of any local authority or other body
Reference: Laxmikanth corporate constituted by law or of any public
institution.
Q.20) Ans: B • Statement 2 is correct: The SPSC is consulted by
Exp: the Governor while framing rules for appointment
• Statement 1 is correct: The SPSC is created directly to judicial service of the state other than the posts
by the Constitution, a JSPSC can be created by an of district judges. In this regard, the concerned
state High Court is also consulted.

DAY 34_IDMP FULL LENGTH TEST 5 21


Contact us : info@onlyias.com

OnlyIAS Nothing Else Visit : dpp.onlyias.in


Contact : +91-7007 931 912

• Statement 3 is correct: The chairperson of a SPSC, the President may direct. There is no provision for
on ceasing to hold office, is eligible for annual reports.
appointment as the chairperson or member of the
UPSC or as the chairperson of any other SPSC, but Reference: Laxmikanth
not for any other employment under the
Government of India or a state.
Q.25) Ans: C
Reference: Laxmikanth Exp:
• Statement 1 is correct: The provision for the
Q.23) Ans: B Special Officer for Linguistic Minorities was made
Exp: through 7th Constitutional Amendment Act, 1956,
• Statement 1 is incorrect: The chairperson or a which inserted Article 350B into the Constitution.
member of a SPSC can be removed from the office • Statement 2 is incorrect: The Constitution does
by the President only in the manner and on the not specify the qualifications, tenure, salaries and
grounds mentioned in the Constitution. They are allowances, service conditions and procedure for
clearly defined in the Constitution and are not like removal of the Special Officer for Linguistic
the manner of a High Court Judge. Minorities.
• Statement 2 is correct: The President can remove • Statement 3 is correct: It is the duty of the Special
the chairperson or any other member of SPSC for Officer to investigate all matters relating to the
misbehaviour. The Constitution has defined safeguards provided for linguistic minorities under
‘misbehaviour’ in this context. The Constitution the Constitution. It includes the provision made
states that the chairman or any other member of a under Article 350A of the Constitution which states
SPSC is deemed to be guilty of misbehaviour, if he that “It shall be the endeavour of every State and
(a) is concerned or interested in any contract or of every local authority within the State to provide
agreement made by the Government of India or adequate facilities for instruction in the mother-
the government of a state, or (b) participates in any tongue at the primary stage of education to
way in the profit of such contract or agreement or children belonging to linguistic minority groups.”
in any benefit therefrom otherwise than as a
member and in common with other members of an Reference: Lakshmikanthan
incorporated company.
• Statement 3 is correct: During the course of
enquiry by the Supreme Court, the governor can Q.26) Ans: A
suspend the concerned chairman or member, Exp:
pending the final removal order of the president on Statement 1 is correct: It becomes a money bill and
receipt of the report of the Supreme Court. thus the President must approve the introduction of
the bill.
Reference: Laxmikanth
Statement 2 is correct: Under Article 3, the President
Q.24) Ans: B must recommend the introduction of this bill as it has
Exp: the potential to significantly affect the federal balance
• Statement 1 is incorrect: The provision for the of power between centre and states.
Special Officer for Linguistic Minorities has been
made under Article 350B of the Constitution which Statement 3 is incorrect: The constitution does not
is in the Chapter IV (Special Directives) under Part distinguish between Private or public bills when it
XVII (Official language) of the Constitution. comes to the requirement of prior Presidential assent.
• Statement 2 is correct: Article 350B(1) provides
that the Special Officer to be appointed by the Statement 4 incorrect: It is only in certain cases, such
President. as alternation of state boundaries which needs prior
• Statement 3 is incorrect: Article 350B(2) provides permission of the President. In other cases, it is not
that the Special Officer shall report to the mandatory.
President upon those matters at such intervals as

DAY 34_IDMP FULL LENGTH TEST 5 22


Contact us : info@onlyias.com

OnlyIAS Nothing Else Visit : dpp.onlyias.in


Contact : +91-7007 931 912

Q.27) Ans: B Statement 1 is incorrect the constitution (Article 76)


Exp: has provided for the office of the Attorney General of
India. He/she is the highest law officer in the country.
Statement 1 is incorrect: He can promulgate an Statement 2 is incorrect the remuneration of the AG is
ordinance only when both the Houses of Parliament not fixed by the constitution. He receives such
are not in session. An ordinance can also be issued remuneration as the President may determine (not
when only one House is in session because a law can fixed by the Parliament).
be passed by both the Houses and not by one House Statement 3 is correct the term of office of the AG is
alone. not fixed by the constitution. Further, the constitution
An ordinance made when both the Houses are in does not contain the procedure and grounds for
session is void. Thus, the power of the President to his/her removal. he/she holds office during the
legislate by ordinance is not a parallel power of pleasure of the president.
legislation.
Extraedge by Onlyias
Statement 2 is correct: This is because a constitutional
A148 under the Constitution provides for an
amendment requires a special majority in both houses
independent office of the Comptroller and Auditor
of Parliament, unlike an ordinary legislation that can be
General of India (CAG). He/she is the guardian of
approved by a simple majority. Moreover, some
the public purse
amendments may require the approval of half of the
states as well with a special majority of parliament.
Source of reference Polity Lazxmikant

Q.28) Ans: C Q.30) Ans: B


Exp: Exp:
Statement 1 is correct The term of office of the AG is
The governor has constitutional discretion in the not fixed by the constitution.He/she is the highest law
following cases: officer in the country. He/she is a part of the Union
executive.
Statement 1 is correct: Seeking information from the Statement 2 is correct the constitution does not
chief minister with regard to the administrative and contain the procedure and grounds for his removal.
legislative matters of the state and Reservation of a bill He/she may be removed by the president any time.
for the consideration of the President. he/she may also quit his/her office by submitting his
resignation to the President.
Statement 2 is incorrect: There is such provision in the Statement 3 is incorrect. He/she must be a person who
constitution. is qualified to be appointed by a judge of the Supreme
court. In other words, he/she must be a citizen of India
Statement 3 is correct: Recommendation for the and must have been a judge of some high court for five
imposition of the President’s Rule in the state. years or an advocate of some high court for ten years
or an eminent jurist, in the opinion of the President.
Statement 4 is correct: While exercising his functions
as the administrator of an adjoining union territory (in
Extraedge by Onlyias
case of additional charge). And also, for Determining
He also represents the Government of India in any
the amount payable by the Government of Assam,
reference made by the President to the SC under
Meghalaya, Tripura and Mizoram to an autonomous
Article 143 of the constitution.
Tribal District Council as royalty accruing from licenses
for mineral exploration.
Source of reference polity Laxmikant

Q.29) Ans: B Q.31) Ans: B


Exp: Exp:

DAY 34_IDMP FULL LENGTH TEST 5 23


Contact us : info@onlyias.com

OnlyIAS Nothing Else Visit : dpp.onlyias.in


Contact : +91-7007 931 912

Statement 1 is correct: Attorney General is not a full- Q.33) Ans: B


time counsel for the government. He does not fall in
the category of government servants. Further, he/she Exp:
is not debarred from private legal practices. • Statement 1 is incorrect: National Commission for
Statement 2 is incorrect: Some of the limitations which Backward Classes (NCBC) was initially constituted
are placed on the Attorney general in order to avoid by the Central Govt by The National Commission
any complication and conflict of duty includes that for Backward Classes Act, 1993. However it is
he/she should not defend accused persons in criminal accorded Constitutional Status and constituted
prosecutions without the permission of GoI. through “The One Hundred and Second
Statement 3 is correct: He/she has the right to speak Amendment constitutional amendment .The One
and to take part in the proceedings of both the houses Hundred and First Amendment)Act, 2016, this
of Parliament or their joint sittings and any committee amendment introduced a national Goods and
of the Parliament of which he may be named a Services Tax (GST) in India from 1 July 2017.
member, without right to vote. He enjoys all the • Statement 2 is correct: The Commission consists
privileges and immunities that are available to a of a Chairperson, Vice-Chairperson and three other
member of Parliament. Members in the rank & pay of Secretary to the
Extraedge by Onlyias Govt of India.
The constitution (Article 76) has provided for the office
of the AG for India. he/she is the highest law officer in Source:
the country. http://www.ncbc.nic.in/User_Panel/UserView.aspx?
TypeID=1025

Source of reference Polity Laxmikant


Q.34) Ans: B
Q.32) Ans: B
Exp: Exp:
Statement 1 is incorrect: An Advocate general of state • Statements 1 and 2 are correct: National
must be a person who is qualified to be appointed a Commission for Backward Classes Chairperson,
judge of a high court (not SC). He must be a citizen of Vice-Chairperson and Members (Conditions of
India and must have held a judicial office for 10 years Service and Tenure) Rules, 2018 provide that The
or have been an advocate of a high court for 10 years. Members of the Commission shall be appointed
Statement 2 is correct: The advocate general is from amongst persons of ability, integrity and
appointed by the Governor. Further the constitution standing who have had a record of selfless service
does not contain procedure and grounds for his to the cause of justice for the socially and
removal. He holds office during the pleasure of the educationally backward classes. The Chairperson
Governor shall be appointed from amongst eminent socio-
Statement 3 is incorrect: The term of office of the political workers belonging to the socially and
advocate general is not fixed by the constitution. The educationally backward classes. At least one other
remuneration of the advocate general is not fixed by Member shall be appointed from amongst
the constitution. He receives such remuneration as the women.
governor may determine. • Statement 3 is incorrect: The Members shall not
be eligible for appointment for more than two
Extraedge by Onlyias terms. Thus the members are eligible for
Article 165 of the constitution provided for the office of appointment for more than one term.
Advocate general of the states. He/she is the highest law
officer in the state. Thus he corresponds to the Attorney Source:
General of India. http://socialjustice.nic.in/writereaddata/UploadFile
/Rules_NCBC_2018.pdf
Page 5
Source of reference Laxmikant polity

DAY 34_IDMP FULL LENGTH TEST 5 24


Contact us : info@onlyias.com

OnlyIAS Nothing Else Visit : dpp.onlyias.in


Contact : +91-7007 931 912

Q.35) Ans: C Q.37) Ans: D


Exp:
Exp: Statement 1 is correct: The National Commission for
Statement 1 is correct: The Chairperson, Vice- Scheduled Castes Consists of a chairperson, a vice-
Chairperson and other Members of the National chairperson and three other members. They are
Commission of the Backward Classes are appointed by appointed by the President by warrant under his hand
the President by warrant under his hand and seal. and seal. Their conditions of service and tenure of
Statement 2 is correct: The Commission had all the office are also determined by the President.
powers of a civil court trying a suit. Statement 2 is correct: The Commission, while
Statement 3 is incorrect: As per Article 342A. (1) the investigating any matter or inquiring into any
President is empowered to specify the socially and complaint, has all the powers of a civil court trying a
educationally backward classes which shall for the suit and in particular in summoning and enforcing the
purposes of this Constitution be deemed to be socially attendance of any person from any part of India and
and educationally backward classes in relation to that examining him on oath.
State or Union territory, as the case may be. Statement 3 is correct: The National Commission for
Scheduled Castes (SCs) is a constitutional body in the
Source: sense that it is directly established by Article 338 of
http://socialjustice.nic.in/writereaddata/UploadFile the Constitution.
/Constitution_102_Amendment.pdf

Source: Indian Polity M. Laxmikanth, Fifth Edition


Q.36) Ans: D

Exp: Q.38) Ans: A


Statement 1 is correct: The Department of Social Exp:
Justice & Empowerment , Ministry of Social Justice and Statement 1 is incorrect: The 65 th Constitutional
Empowerment is entrusted with the empowerment of Amendment Act of 1990 provided for the
the disadvantaged and marginalized sections of the establishment of a high level multi-member National
society. The target groups of the Ministry are: Commission for SCs and STs in the place of a single
Scheduled Castes, Other Backward Classes ,Senior Special Officer for SCs and STs.
Citizens ,Victims of Substance Abuse ,Denotified, The 89 th Constitutional Amendment Act of 2003
Nomadic and Semi-Nomadic Tribes, Beggars, bifurcated the combined National Commission for SCs
Transgender. and STs into two separate bodies, namely, National
Statement 2 is correct: The Constitution has neither Commission for Scheduled Castes (under Article 338)
specified the Backward Classes (BCs) nor used a single and National Commission for Scheduled Tribes (under
uniform expression to characterise the BCs. The Article 338-A).
expression ‘BCs’ means such backward classes of The separate National Commission for SCs came into
citizens other than the SCs and the STs as may be existence in 2004.
specified by the Central Government. Statement 2 is correct: The Commission submits its
Statement 3 is Correct: Article 338B of the annual report to the President of India.
Constitution of India provides that there shall be a Statement 3 is correct: The Central government and
Commission for the socially and educationally the state governments are required to consult the
backward classes to be known as the National Commission on all major policy matters affecting the
Commission for Backward Classes. SCs.
Statement 4 is correct: Besides the members of the
Source: Scs the Commission has to investigate all matters
http://socialjustice.nic.in/UserView/index?mid=766 relating to the constitutional and other legal
61 safeguards of the Anglo-Indian Community and report
M. Laxmikanth, Fifth Edition : Page 945, Specification to the President upon their working. Thus provides
of Classes safeguards against the exploitation of the community.

DAY 34_IDMP FULL LENGTH TEST 5 25


Contact us : info@onlyias.com

OnlyIAS Nothing Else Visit : dpp.onlyias.in


Contact : +91-7007 931 912

Source: Indian Polity M. Laxmikanth, Fifth Edition • Statement 1 is Incorrect: Originally the Election
commission was not a multi member body it was a
single member body. For the first time two
Q.39) Ans: B additional Commissioners were appointed on 16th
October 1989 but they had a very short tenure till
Exp: 1st January 1990. Later, on 1st October 1993 two
• Statement 1 is incorrect: The National additional Election Commissioners were
Commission for Scheduled Tribes (NCST) was appointed. The concept of multi-member
established by amending Article 338 and inserting Commission has been in operation since then, with
a new Article 338A in the Constitution through the decision making power by majority vote.
Constitution (89th Amendment) Act, 2003.The • Statement 2 is Correct: In Mohinder Singh Gill &
term of office of Chairperson, Vice-Chairperson Anr. vs The Chief Election Commissioner and
and each member is three years from the date of Others (December 2, 1977), the Supreme Court
assumption of charge. ruled that “Article 324, on the face of it, vests vast
• Statement 2 is correct: The Chairperson has been functions in the Commission, which may be powers
given the rank of Union Cabinet Minister, and the or duties, essentially administrative, and
Vice-Chairperson that of a Minister of State and marginally, even judicative or legislative”
other Members have the rank of a Secretary to the Source:
Government of India. https://www.google.com/amp/s/indianexpress.com/a
rticle/what-is/what-is-article-324-in-the-indian-
Source: https://ncst.nic.in/content/introduction constitution-5729800/lite/

Q.40) Ans: A Q.42) Ans: D

Exp: Exp:
Statement 1 is correct: The National Commission for • Statement 1 is Correct: Election commission,
Scheduled Tribes, shall consist of a Chairperson, Vice- under Article 324, has to ensure free and fair
Chairperson and three other Members and the election. Free and fair election, although objective
conditions of service and tenure of office of the of Election commission, is also a Basic Structure
Chairperson, Vice-Chairperson and other members so • Statement 2 is Correct: T.N. Seshan v. Union of
appointed shall be such as the President may India, the Supreme Court unanimously upheld the
determine. constitutionality of the. Chief Election
Statement 2 is correct: The Commission is vested with commissioner & other Election commissioner
powers of a civil court having authority to: Summon (condition of service) Amendment Act 1994
and enforce attendance of any person and examine on equating the status, powers and authority of the
oath; Discovery & production of any documents, two Election Commissioners with that of the CEO.
Receive evidence on affidavits; Requisition any public • Statement 3 is Correct: the tenure of office of
record or copy thereof from any court or office; Issue other Election Commissioners and the Regional
Commissions for examination of witnesses and Commissioners is also free of the political
documents; and Any matter which President, by rule, executive control so far, none of them can be
may determine. removed from office except on the
Statement 3 is incorrect: The Union Minister of Tribal recommendation of the CEC.
Affairs does not serve as an Ex-officio Vice-Chairperson Source:
of the Commission. http://www.legalserviceindia.com/legal/article-2032-
election-commission-of-india-articles-324-to-329-
.html
Q.41) Ans: B

Exp: Q.43) Ans: C

DAY 34_IDMP FULL LENGTH TEST 5 26


Contact us : info@onlyias.com

OnlyIAS Nothing Else Visit : dpp.onlyias.in


Contact : +91-7007 931 912

Exp: candidate. It includes every process issued after


• Statement 1 is Correct: Clause (6) of Article 324 the issuance of the Notification for holding the
provides that the President, or the Governor of a election
State shall, when so requested by the Election • Statement 2 is Incorrect: Ram Deo Bhandari v.
Commission, make available to the Election Election Commission,the Supreme Court held that
Commission or to a Regional Commissioner such the Election Commission was free to take such
staff as may be necessary for the discharge of the steps as it considered necessary to ensure a free
functions conferred on the Election Commission. and fair poll, but, it would not withhold the
• Statement 2 is Incorrect: The Commission shall elections to the Legislative Assembly of a State on
have power to recommend disciplinary action, to the ground that the said Government had failed to
the competent authority, against the officers for complete the process of issuance of photo identity
insubordination or dereliction of duty while on cards by the deadline prescribed by the
election duty. Hence it can only recommend Commission.
disciplinary action & not take action by itself • Statement 3 is Incorrect: Section 22 of the
• Statement 3 is Correct: Election commission has Representation of the People Act, 1950 empowers
been given power to superintendence, direction the Electoral Registration Officer of a constituency
and control of the preparation of electoral rolls for to delete the name of a person from the Electoral
all elections to both Parliament and to the Roll on certain grounds. It has been held that such
Legislature of every State deletion must be done only after giving to the
Source: person concerned meaningful opportunity of
http://www.legalserviceindia.com/legal/article-2032- hearing and after following requisite procedure.
election-commission-of-india-articles-324-to-329- Source:
.html http://www.legalserviceindia.com/legal/article-2032-
election-commission-of-india-articles-324-to-329-
.html
Q.44) Ans: D

Exp: Q.46) Ans: C


The power conferred on the Commission under Article
324 (1) is subjected to two limitations, namely Exp:
• Statement 1 is correct: When Parliament or any • Both Statements 1 and 2 are correct: The Finance
State Legislature has made a valid law relating to Commission is constituted by the President under
or in connection with elections, the Commission article 280 of the Constitution, mainly to give its
shall act in conformity with such law. recommendations on distribution of tax revenues
• Statement 2 is Correct: The Commission while between the Union and the States and amongst
exercising power shall conform to the rule of law, the States themselves. Two distinctive features of
act bona fide and be amenable to the norms of the Commission’s work involve
natural justice. o Redressing the vertical imbalances
Source: between the taxation powers and
http://www.legalserviceindia.com/legal/article-2032- expenditure responsibilities of the
election-commission-of-india-articles-324-to-329- centre and the States respectively
.html ,and
o Equalization of all public services
across the States.
Q.45) Ans: C
Source:
Exp: https://fincomindia.nic.in/ShowContentOne.aspx?id=
• Statement 1 is Correct: The word elections in 8&Section=1
Article 324 includes the entire process of election,
which embraces many steps some of which have
an important bearing on the process of choosing a Q.47)

DAY 34_IDMP FULL LENGTH TEST 5 27


Contact us : info@onlyias.com

OnlyIAS Nothing Else Visit : dpp.onlyias.in


Contact : +91-7007 931 912

Ans: B Source:
Exp: https://fincomindia.nic.in/ShowContentOne.aspx?id=
It is the duty of the Commission to make 8&Section=1
recommendations to the President in following
matter -
• Statement 1 is Correct: For the distribution Q.49) Ans: A
between the Union and the States of the net
proceeds of taxes which are to be, or may be, Exp:
divided between them and the allocation between • Statement 1 is Correct while Statement 2 is
the States of the respective shares of such Incorrect
proceeds; • The recommendations of the Finance Commission
• Statement 2 is Incorrect and Statement 3 is are implemented as under:-
Correct: the principles which should govern the o Those to be implemented by an order
grants-in-aid of the revenues of the States out of of the President:
the Consolidated Fund of India. The measures ▪ The recommendations
needed to augment the Consolidated Fund of a relating to distribution of
State to supplement the resources of the Union Taxes and Duties and
Panchayats and the municipality in the State on the Grants-in-aid fall in this
basis of the recommendations made by the category.
Finance Commission of the State. o Those to be implemented by executive
• Statement 4 is correct: any other matter referred orders:
to the Commission by the President in the interests ▪ Other recommendations to be
of sound finance made by the Finance
Source: Commission, as per its Terms
https://fincomindia.nic.in/ShowContentOne.aspx?id= of Reference
8&Section=1 Source:
https://fincomindia.nic.in/ShowContentOne.aspx?id=
8&Section=1
Q.48) Ans: B

Exp: Q.50) Ans: D


• Statement 1 is Incorrect & Statement 2 is
Correct: As per the provisions contained in the Exp:
Finance Commission [Miscellaneous Provisions] • Disqualifications for being a member of the
Act, 1951 and The Finance Commission (Salaries & Commission
Allowances) Rules, 1951, the Chairman of the • Statement 1 is Correct: a person shall be
Commission is selected from among persons who disqualified for being appointed as, or for being, a
have had experience in public affairs, and the four member of the Commission, --
other members are selected from among persons o if he is of unsound mind;
who- o if he is an undischarged insolvent;
o are, or have been, or are qualified to o if he has been convicted of an offence
be appointed as Judges of a High involving moral turpitude;
Court; or o if he has such financial or other
o Have special knowledge of the interest as is likely to affect
finances and accounts of Government; prejudicially his functions as a member
or of the Commission
o Have had wide experience in financial • Statement 2 is Correct: Before appointing a
matters and in administration; or person to be a member of the Commission, the
o Have special knowledge of economics President shall satisfy himself that that person will
have no such financial or other interest as is likely

DAY 34_IDMP FULL LENGTH TEST 5 28


Contact us : info@onlyias.com

OnlyIAS Nothing Else Visit : dpp.onlyias.in


Contact : +91-7007 931 912

to affect prejudicially his functions as a member of be, ought on any such ground to be
the commission removed.
Source: o is adjudged an insolvent; or
https://fincomindia.nic.in/ShowContent.aspx?uid1=2 o engages during his term of office in any
&uid2=3&uid3=0&uid4=0 paid employment outside the duties of
his office; or
o is, in the opinion of the President, unfit
Q.51) Ans: D to continue in office by reason of
infirmity of mind or body.
Exp: • Statement 2 is Correct: If the Chairman or any
A/c to THE FINANCE COMMISSION (MISCELLANEOUS other member of a Public Service Commission is or
PROVISIONS) ACT, 1951 becomes in any way concerned or interested in any
• Statement 1 and Statement 2 are Correct: contract or agreement made by or on behalf of the
Procedure and powers of the Commission Government of India or the Government of a State
The Commission shall determine their procedure and or participates in any way in the profit thereof or in
in the performance of their functions shall have all the any benefit or emolument arising therefrom
powers of a civil court under the Code of Civil otherwise than as a member and in common with
Procedure, 1908 (5 of 1908), while trying a suit in the other members of an incorporated company,
respect of the following matters, namely:-- he shall,be deemed to be guilty of misbehaviour
o summoning and enforcing the Source: https://www.upsc.gov.in/about-
attendance of witnesses; us/constitutional-provisions/article-317-removal-and-
o requiring the production of any suspension-member-public-service-commission
document;
o requisitioning any public record from
any court or office Q.53) Ans: C

Exp:
• Statement 3 is Correct: The Commission shall have • Statement 1 is Incorrect & Statement 2 is correct:
power to require any person to furnish information In the case of the Union Commission or a Joint
on such points or matters as in the opinion of the Commission, the President and, in the case of a
Commission may be useful for, or relevant to, any State Commission, the Governor of the State may
matter under the consideration of the by regulations
Commission. o determine the number of members of
Source: the Commission and their conditions
https://fincomindia.nic.in/ShowContent.aspx?uid1=2 of service; and
&uid2=3&uid3=0&uid4=0 o make provision with respect to the
number of members of the staff of the
Commission and their conditions of
Q.52) Ans: B service:
• Provided that the conditions of service of a
Exp: member of a Public Service Commission shall not
• Statement 1 is Incorrect: Removal of be varied to his disadvantage after his
member/chairperson of UPSC appointment.
o Misbehaving - after the Supreme • Statement 3 is correct: The expenses of the Union
Court, on reference being made to it or a State Public Service Commission, including any
by the President, has, on inquiry held salaries, allowances and pensions payable to or in
in accordance with the procedure respect of the members or staff of the Commission,
prescribed in that behalf under article shall be charged on the Consolidated Fund of India
145, reported that the Chairman or or, as the case may be, the Consolidated Fund of
such other member, as the case may the State

DAY 34_IDMP FULL LENGTH TEST 5 29


Contact us : info@onlyias.com

OnlyIAS Nothing Else Visit : dpp.onlyias.in


Contact : +91-7007 931 912

Source: https://www.upsc.gov.in/about- • Statement 2 is Correct: The Union Public Service


us/constitutional-provisions Commission or the State Public Service
Commission, as the case may be, shall be consulted
Q.54) Ans: C - on any claim for the award of a pension in respect
Exp: of injuries sustained by a person while serving
• On ceasing to hold office under the Government of India or the Government
o the Chairman of the Union Public Service of a State in a civil capacity, and any question as to
Commission shall be ineligible for further the amount of any such award
employment either under the Government of • Source: https://www.upsc.gov.in/about-
India or under the Government of a State; us/constitutional-provisions
• Statement 1 and statement 4 is correct:the
Chairman of a State Public Service Commission
shall be eligible for appointment as the Chairman Q.56) Ans: D
or any other member of the Union Public Service
Commission or as the Chairman of any other State Exp:
Public Service Commission, but not for any other • The modern-day role of the Comptroller and
employment either under the Government of India Auditor General of India has developed through
or under the Government of a State. the practice and traditions that were followed
• Statement 2 is correct: a member other than the during colonial British India
Chairman of a State Public Service Commission • Statement 1 is Correct: Sir Edward Drummond was
shall be eligible for appointment as the Chairman appointed as the first Auditor General of India in
or any other member of the Union Public Service November 1860 and his authority derived from the
Commission or as the Chairman of that or any Government of India Act, 1858
other State Public Service Commission, but not for • Statement 2 is Correct: After a few more years, the
any other employment either under the Montford Reforms of 1919 made the office of CAG
Government of India or under the Government of independent from the sovereign.
a State • Statement 3 is Correct: The powers of CAG were
• Statement 3 is incorrect: a member other than the reinforced by the Government of India Act, 1935 by
Chairman of the Union Public Service Commission provisioning for Provincial Auditor-Generals in the
shall be eligible for appointment as the Chairman federal setup of British India. The Government of
of the Union Public Service Commission or as the India (Audit and Accounts) Order, 1936 laid down
Chairman of a State Public Service Commission, but the conditions of service of the Auditor General,
not for any other employment either under the and also defined his duties and powers concerning
Government of India or under the Government of audits and reports
a State • Source:
Source: https://www.upsc.gov.in/about- https://www.google.com/amp/s/blog.ipleaders.in
us/constitutional-provisions /comptroller-auditor-general-india/amp/

Q.55) Ans: C Q.57) Ans: B

Exp: Exp:
• Statement 1 is Correct: An Act made by Parliament • Statement 1 is Incorrect: The Comptroller and
or, as the case may be, the Legislature of a State Auditor General (Duties, Powers and Conditions of
may provide for the exercise of additional Service) Act 1971, made the office responsible for
functions by the Union Public Service Commission accounting and auditing of both the Central and
or the State Public Service Commission as respects the State governments in pursuance of the quasi-
the services of the Union or the State and also as federal nature of the country. However, in 1976,
respects the services of any local authority or other the CAG was relieved from its accounting functions
body corporate constituted by law or of any public • Statement 2 is Correct: Article 148 - The salary and
institution. other conditions of the CAG’s services shall be

DAY 34_IDMP FULL LENGTH TEST 5 30


Contact us : info@onlyias.com

OnlyIAS Nothing Else Visit : dpp.onlyias.in


Contact : +91-7007 931 912

specified in the Second Schedule of the • Statement 1 is Incorrect: CAG derives its mandate
Constitution until determined by the Parliament from Constitution and the CAG's (Duties, power
Source: and conditions of service) Act 1971
https://www.google.com/amp/s/blog.ipleaders.in/co • Statement 2 is Correct: The importance of Duties
mptroller-auditor-general-india/amp/ and Powers stemming from the Article 149 of the
Constitution is manifested in following observation
of the apex court in a 2014 judgment: “…CAG,
Q.58) Ans: A therefore, is exercising constitutional powers and
duties in relation to the accounts, while the High
Exp: Court under Article 226 of the Constitution, so also
• Statement 1 is Correct: The CAG is appointed by the Supreme Court under Article 32 of the
the President of India by warrant under his hand Constitution, is exercising judicial powers. Duties
and seal and powers conferred by the Constitution on the
• Statement 2 is Incorrect : Section IV of the Third CAG under Article 149 cannot be taken away by the
Schedule of the Constitution lays down the form of Parliament, being the basic structure of our
oath or affirmation for both the Judges of the Constitution, like Parliamentary democracy,
Supreme Court as well as the Comptroller and independence of judiciary, rule of law, judicial
Auditor General of India. review, unity and integrity of the country, secular
• Statement 3 is Correct: Article 150 says the and federal character of the Constitution, and so
President of India shall prescribe to the CAG the on.
form in which the accounts of the Union and the Source:
States shall be maintained and kept https://www.google.com/amp/s/m.thewire.in/article/
• Source: law/courts-cag-and-the-process-of-conducting-an-
https://www.google.com/amp/s/blog.ipleaders.in audit/amp
/comptroller-auditor-general-india/amp/

Q.61) Ans: B
Q.59) Ans: B
Exp:
Exp: • Statement 1 is Incorrect: Originally, Article 338 of
• Statement 1 is Correct: The CAG is authorised to the Constitution provided for the appointment of a
audit the accounts and expenditures from the Special Officer for Scheduled Castes (SCs) and
Consolidated Fund of India, and of States and those Scheduled Tribes (STs) to investigate all matters
Union Territories which have legislative relating to the constitutional safeguards for the SCs
assemblies. and STs and to report to the President on their
• Statement 2 is Incorrect: CAG also audits all working.
expenditures from the Contingency Fund of India o In 1978, the Government (through a
(Article 267) and Public Account of India (Clause 2 Resolution) set up a non-statutory
of Article 266) as well as those of the states multi member Commission for SCs and
• Statement 3 is Correct: On request from the STs; the Office of Commissioner for SCs
President or Governor, CAG also audits the reports and STs also continued to exist.
of the local bodies o In 1987, the Government (through
Source: another Resolution) modified the
https://www.google.com/amp/s/blog.ipleaders.in/co function of commission and renamed
mptroller-auditor-general-india/amp/ it as the National Commssion for SCs
and STs
• Statement 2 is Correct: 89th Constitutional
Q.60) Ans: A Amendment Act of 2003 bifurcated the combined
National Commission for SCs and STs into two
Exp: separate bodies, namely, National Commission for
Scheduled Castes (under Article 338) and National

DAY 34_IDMP FULL LENGTH TEST 5 31


Contact us : info@onlyias.com

OnlyIAS Nothing Else Visit : dpp.onlyias.in


Contact : +91-7007 931 912

Commission for Scheduled Tribes (under Article • Statement a, b and d is incorrect: Functions of
338-A). National commission for scheduled Tribe
o The separate National Commission for
SCs came into existence in 2004
• Source: OnlyIAS polity Booklet o To investigate and monitor matters
relating to the constitutional and other
legal safeguards for the STs
Q.62) Ans: B o Inquire specific complaints relating to
deprivation of rights of ST’s
Exp: o Participate and advice on the planning
• Statement 1 is Incorrect: To identify and process of socioeconomic
distinguish these communities, the Chanda development of the STs and to
Committee in the year 1960 had laid down 5 evaluate the progress of their
standards to include any community/caste in the development under the Union or a
tribal group state
• Statement 2 is Correct: According to the 2011 o To present to the President, annually
Census, the Scheduled Tribes account for 104 and at such other times as it may deem
million representing 8.6% of the country’s fit, reports upon the working of those
population safeguards
• Statement 3 is Incorrect: Ministry of social justice o To make recommendation for
and empowerment coordinate all activities related effective implementation of
to schedule caste while ministry of Tribal affairs safeguards
coordinates all activities related to schedule tribe o To discharge such other functions in
• Source: OnlyIAS polity Booklet relation to the protection, welfare and
development an advancement of the
ST’s.
Q.63) Ans: C

Exp: • Statement c is Correct: The President may, with


• Statement 1 is Correct: The complaints should respect to any State or Union territory, and where
disclose his full identity and give his full address it is a state, after consultation with the Governor
and should sign the representation. thereof by public notification, specify the tribes or
• Statement 2 is Incorrect: The complaint should be tribal communities or parts of or groups within
directly addressed to the Chairperson/Vice- tribes or tribal communities which shall, for the
Chairperson/Secretary, National Commission for purposes of this constitution, is deemed to be
Scheduled Tribes, New Delhi or the heads of its Scheduled Tribe
State Offices. Complaints should be legibly written Source: OnlyIAS polity Booklet
or typed and, where necessary, supported by
authenticated documents.
• Statement 3 is Incorrect: No action will be taken Q.65) Ans: D
on matters which are sub judice. Hence sub judice
matters need not be referred to the Commission as Exp:
complaint(s). Cases pending in courts or cases in • Statement a, b and c is incorrect: in accordance
which a court has already given its final verdict with Clause (8) of Article 338 of the Constitution,
need not be taken up afresh with the Commission. while investigating in a matter or in inquiring into
Source: https://ncst.nic.in/content/procedure-inquiry any complaint the Commission shall have all the
powers of civil court trying a suit and in particular
in respect of the following matters, namely:-
Q.64) Ans: C o summoning and enforcing the
attendance of any person from any
Exp:

DAY 34_IDMP FULL LENGTH TEST 5 32


Contact us : info@onlyias.com

OnlyIAS Nothing Else Visit : dpp.onlyias.in


Contact : +91-7007 931 912

part of India and examining him on a case of contempt of court against a person.
oath; Before such a plea can be filed, the Attorney
o requiring the discovery and General must sign off on the complaint,
production of any document; determining if it requires the attention of the court
o receiving evidence on affidavits; at all. However, when the court itself initiates a
o requisitioning any public record or contempt of court case, the AG’s consent is not
copy thereof from any court or office; required. This is because the court is exercising its
o issuing commissions for the inherent powers under the Constitution to punish
examination of witnesses and for contempt and such Constitutional powers
documents; cannot be restricted because the AG declined to
o any other matter which the President grant consent
may, by rule, determine. • Statement 3 is Incorrect: If the Attorney General
Source: https://indiankanoon.org/doc/50707649/ denies consent, the matter all but ends. The
complainant can, however, separately bring the
issue to the notice of the court and urge the court
Q.66) Ans: A to take suo motu (on its own motion) cognizance.
If the court does take suo motu cognizance, as it
Exp: did in the case of Prashant Bhushan, the consent of
• Statement 1 is Incorrect: Article 338 B in the the senior-most law officer is not required.
Constitution, provides for a National Commission • Source:
for Backward Classes with a Chairperson, Vice- https://www.google.com/amp/s/indianexpress.co
Chairperson and three other members, all of m/article/explained/consent-for-contempt-
whom shall be appointed by the President of India. attorney-general-for-india-k-k-venugopal-rachita-
Their tenure and conditions of service will also be taneja/lite/
decided by the President through rules.
• Statement 2 is Incorrect: Till 2018, the National
commission for Scheduled castes was also required Q.68) Ans: A
to discharge functions with regard to the other
backward classes (OBCs). It was relieved from this Exp:
responsibility by the 102nd Amendment Act of • Statement 1 is Correct: His Term of office not fixed
2018. by the Constitution and Constitution has not
• Statement 3 is correct: The Kalelkar Commission, prescribed any procedure for his removal. He holds
set up in 1953, was the first to identify backward office during the pleasure of the President. He
classes other than the Scheduled Castes and receives remuneration as President may
Scheduled Tribes at the national level. determine
Source: OnlyIAS polity Booklet • Statement 2 is Correct: He has the Right to speak
and take part in the proceedings of both the
houses of parliament, joint sitting, or any
Q.67) Ans: C committee proceedings. But he does not have the
right to vote
Exp: • Statement 3 is Incorrect: He can't defend an
• Statement 1 is Correct: In the case of the Supreme accused person, only in criminal proceedings
Court, the Attorney General or the Solicitor without the permission of the Government of
General, and in the case of High Courts, the India.
Advocate General, may bring in a motion before • Source: OnlyIAS polity Booklet
the court for initiating a case of criminal contempt.
However, if the motion is brought by any other Q.69) Ans: D
person, the consent in writing of the Attorney
General or the Advocate General is required. Exp:
• Statement 2 is Correct : The AG’s consent is
mandatory when a private citizen wants to initiate

DAY 34_IDMP FULL LENGTH TEST 5 33


Contact us : info@onlyias.com

OnlyIAS Nothing Else Visit : dpp.onlyias.in


Contact : +91-7007 931 912

• Statement 1 is Correct: Originally, the Constitution • Statement 2 is correct: Article 25 of the


of India did not make any provision with respect to Constitution guarantees rights to individuals to
the Special Officer for Linguistic Minorities. religious conscience and to freely profess,
o States Reorganisation Commission practice, and propagate a religion of their
(1953–55) made a recommendation in choice. It protects the rights of the individuals.
this regard. Article 26 of the Constitution deals with the
• Statement 2 is Correct: The Seventh Constitutional rights of the religious denominations or the
Amendment Act of 1956 inserted a new Article sections of individuals.
350-B in Part XVII of the Constitution • Statement 3 is correct: It was in the S. R.
• Statement 3 is Correct: The Constitution does not Bommai Case that the Supreme Court held that
specify the qualifications, tenure, salaries and Secular activities need to be separated from
allowances, service conditions and procedure for religion. Religion is a matter of personal faith
removal of the Special Officer for Linguistic and hence cannot be mixed with secular
Minorities activities.

Source:
Q.70) Ans: C https://www.thehindu.com/news/national/supreme-
Exp: court-notice-to-govt-on-plea-for-uniform-laws-of-
• Article 148 of the constitution provides for an succession/article34039787.ece ; M. Laxmikanth.
independent office of the CAG. It is the supreme
audit institution of India. Q.72) Ans: A
• Statement 1 is Correct: He is appointed by the
President of India by a warrant under his hand and Exp:
seal. His Salary and other service conditions are
determined by the Parliament. • Statement 1 is incorrect: A law seeking
• Statement 2 is Incorrect: No minister can preferential treatment on domicile related
represent the CAG in parliament grounds cannot be enacted by the state
Statement 3 is Correct: CAG can be removed by the government. As per Article 16(3), such laws
President on the same grounds and in the same can be enacted only by the Parliament. Hence,
manner as a judge of the Supreme Court i.e he can be laws seeking to provide local reservations can
removed by the President on the basis of a resolution be declared as unconstitutional on the above
passed to that effect by both the Houses of Parliament grounds.
with special majority, either on the ground of proved • Statement 2 is correct: Nagraj case (2006) was
misbehaviour or incapacity important as the Supreme Court allowed
reservation to be extended to the SCs and STs
in case of promotions. However, it was
subjected to the below mentioned three
Q.71) Ans: B conditions: Presence of quantifiable data on
the backwardness of SCs and STs is available;
Exp: there is inadequate representation; and there
is no adverse impact on the overall efficiency
• Statement 1 is incorrect: The subject personal of the administration.
laws are mentioned in the Concurrent List of
the Constitution. Hence, Parliament does not Source: https://www.hindustantimes.com/india-
have exclusive jurisdiction over this domain. news/is-haryana-s-new-reservation-law-
The matters regarding the Uniform Civil Code constitutionally-tenable-101615180936227.html ; M.
are mentioned under the Directive Principles Laxmikanth
under Article 44 of the Constitution. Presently,
different personal laws govern the citizens of Q.73) Ans: B
different religious communities.
Exp:

DAY 34_IDMP FULL LENGTH TEST 5 34


Contact us : info@onlyias.com

OnlyIAS Nothing Else Visit : dpp.onlyias.in


Contact : +91-7007 931 912

• Option B is correct: IUCN or the International Centres. It also funds the expenditures of the
Union for the Conservation of Nature is National Health Mission.
composed of both the government and the • Statement 2 is incorrect: This is a fund
civil society organizations. It assesses the maintained under the Public Account of India.
global status of the natural world along with The proceeds of the fund are catered to from
the measures needed to safeguard them. It the Health and Education cess levied under the
maintains a red data book that mentions the Finance Act, 2007.
lists of the endangered species. It has been
classified into various groups depending upon Source:
the extent of threat faced. These are Extinct, https://pib.gov.in/PressReleasePage.aspx?PRID=1703
Extinct in the wild, Critically Endangered, 736
Endangered, Vulnerable, Near Threatened,
Least Concern, Data Deficient. Extinct status is Q.75) Ans: A
accorded to species where it is known that
there is no living individual left. Extinct in the Exp:
Wild is accorded to those who are known to be
surviving only in the artificial environment, like • Statement 1 is incorrect: The Unlawful
in captivity. The three categories of Critically Activities (Prevention) Act, 1967 was enacted
Endangered, Endangered, and Vulnerable are to effectively prevent the terrorists and the
grouped under the threatened category. unlawful activities of the individuals and
Critically Endangered organisms are known to associations. Cases under this Act can be
have less than 50 mature individuals. investigated both by the National Investigation
Endangered species are those which have less Agency and the state police.
than 250 surviving individuals while the • Statement 2 is incorrect: As per this Act, the
Vulnerable species have less than 10,000 living Central government is empowered to impose
individuals. a ban on an organization by declaring them as
• The classification of the above organisms is unlawful. The ban can be done both on the
done as under: individuals as well as the organizations dealing
o Forest Elephants: Critically with terror related activities.
Endangered • Statement 3 is correct: A recent amendment
o Asian Elephants: Endangered of the Act in 2019 extended the ban on both
o Dugong: Vulnerable the individuals and the organization by
o Olive Ridley Turtle: Vulnerable declaring them as terrorists. Earlier only the
organizations could be banned by declaring
Source: them as terrorist organizations but now even
https://www.iucn.org/news/species/202103/african- individuals can be declared as terrorists under
elephant-species-now-endangered-and-critically- the provisions of this Act.
endangered-iucn-red-list ; Shankar IAS.
Source:
Q.74) Ans: A https://www.mha.gov.in/sites/default/files/CTCR_Unl
awfulActivities_06092019.pdf
Exp:
Q.76) Ans: B
• Statement 1 is correct: Prime Minister
Swasthya Suraksha Nidhi (PMSSN) is a single Exp:
non-lapsable reserve fund that seeks to fund
the health-related expenditures. The fund is • Option B is correct: Port Anjouan is located in
maintained and administered by the Ministry the Comoros Islands near Madagascar. INS
of Health and Family Welfare. It seeks to Jalashwa recently arrived at this port. Port
provide funding to the Ayushmann Bharat- PM Ehoala is located in the Madagascar and was
Jan Arogya Yojana and the Health and Wellness visited by INS Jalashwa. Mongla port is located

DAY 34_IDMP FULL LENGTH TEST 5 35


Contact us : info@onlyias.com

OnlyIAS Nothing Else Visit : dpp.onlyias.in


Contact : +91-7007 931 912

in Bangladesh and was visited by the first energy density. It offers a greater range of
Indian navy ship. distance covered (400 km approx..) per
battery as against the Lithium-ion batteries. It
Source: Atlas can be recycled and be put into industrial use.
The disadvantage of these batteries is that
Q.77) Ans: D they cannot be recharged.
Exp:
Source:
• Statement 1 is incorrect: A recent research has https://indianexpress.com/article/explained/indian-
indicated that heat waves in India have been oil-corporation-phinergy-aluminium-batteries-
attributed to the Quasi-Resonant 7235560/
Amplification. Quasi-Resonant Amplification
occurs due to the warming of the Arctic region. Q.79) Ans: C
It has been associated with global warming. It
has been observed that any increase in the Exp:
temperature of the Arctic has led to rise of
heat waves in India, even in the states of • Statement 1 is incorrect: United Nations
Andhra Pradesh and Telangana. Hence, Quasi- General Assembly recently adopted the year
Resonant Amplification is not associated with 2023 as the international year of Millets.
the warming of temperature and tropical Millets are the coarse grains and are a gluten-
latitudes. It is rather associated with the free alternative. India is the largest producer of
warming of the Arctic region. Millets with a global share of production
• Statement 2 is incorrect: Instances of Heat amounting to around 40%.
waves attributed to Quasi-Resonant • Statement 2 is incorrect: Millets are classified
Amplification have been witnessed in India. A as the Major and the Minor millets in
number of extreme weather events of the past accordance with the size of the grain. Major
have been directly or indirectly associated with millets like Jowar, Bajra, Ragi are essentially
QRA, this includes, 2003 European Heatwave, grown during the Kharif season. Unlike rice
Pakistan floods of 2010, etc. these are less water intensive and drought and
heat resistant crops. They can easily be found
Source: https://www.thehindu.com/sci-tech/energy- in areas with less than 350 mm of rainfall.
and-environment/arctic-warming-is-causing-heat- • Statement 3 is correct: Rajasthan accounts for
waves-in-india-study/article34157092.ece the largest area under the cultivation of Millets
followed by Maharashtra and Karnataka.
Q.78) Ans: C
Source:
Exp: https://www.thehindu.com/news/international/unga-
unanimously-adopts-india-sponsored-resolution-
• Option 2, 3, and 4 is correct: Indian Oil declaring-2023-as-international-year-of-
Corporation has entered a joint Venture with millets/article33986118.ece
the Israel based battery technology startup
Phinergy for the development of the Q.80) Ans: D
aluminum-air based technology system for the
development of the batteries for the electric Exp:
vehicles and the stationary storage systems.
Aluminium-air batteries make use of oxygen in • Option D is correct: Ken-Betwa River linking
the air which reacts with the aluminium project is the first river linking project under
hydroxide solution to oxide the aluminium and the National River Linking Project. It was
produce electricity. Aluminium-air batteries signed between the Union Minister of Jal
offer a number of advantages over the Shakti and the chief Ministers of Uttar Pradesh
Lithium-ion batteries. Aluminum air batteries and Madhya Pradesh. The projects aim to
are cheaper and lighter and have greater

DAY 34_IDMP FULL LENGTH TEST 5 36


Contact us : info@onlyias.com

OnlyIAS Nothing Else Visit : dpp.onlyias.in


Contact : +91-7007 931 912

transfer the waters of river Ken to Betwa, between-indus-commissioners-of-india-pakistan-


which are both the tributaries of river Yamuna. underway/article34138593.ece
The implementation of the project would be
undertaken by a special purpose vehicle Q.82) Ans: D
created for this purpose. The construction of
Daudhan and Orr Dam have been envisaged in Exp:
this project in different phases.
• Statement 1 is incorrect: Quad or the
Source: Quadrilateral Security Dialogue is a grouping
https://www.thehindu.com/news/national/apex- of the USA, India, Japan, Australia with similar
body-on-river-linking-in-pipeline/article30976264.ece geopolitical issues along the Indo-Pacific
region. It is not a military grouping like NATO
Q.81) Ans: C and hence is not mandated to militarily
support the member countries during a war.
Exp: The latest development in the Quad has been
the Summit level meeting in 2021. It centered
• Statement 1 is correct: Indus Water Treaty around issues regarding Vaccine, Climate
was signed in 1960 between India and Change, and Critical and emerging
Pakistan and was brokered by the World Bank. technologies.
The treaty envisaged the creation of the • Statement 2 is incorrect: The idea of Quad was
Permanent Indus Commission with initiated in 2004 after the Indian Ocean
representatives from each country to ensure Tsunami when the four countries came
the implementation of the treaty. It allocated together. Even after the emergence of the idea
waters of the eastern rivers (Sutlej, Beas, and of the Indo Pacific region in 2007, the grouping
Ravi) to India while the waters of the western remained dormant till 2017. It was on the
rivers (Indus, Jhelum, and Chenab) were sidelines of the East Asia Summit in 2017 in
allocated to Pakistan. The treaty also provided the Philippines that the revival of the grouping
for the building of canals, dams, etc. The occurred. This was followed by the Quad
Tarbela Dam was envisaged on the Indus River meeting in 2019 in Washington and the
while the Mangla Dam on the Jhelum River. summit level talks in 2021.
• Statement 2 is correct: The waters of the
western rivers have been allocated to Pakistan, Source:
yet India is allowed to develop the run off the https://www.financialexpress.com/defence/quad-
river projects on these rivers in accordance summit-2021-why-is-china-rattled/2212544/
with the technical specifications provided in
the agreement. India has undertaken the Q.83) Ans: C
construction of several such projects like the
Kishanganga Hydroelectric project on the river Exp:
Jhelum and the Ratle Hydroelectric project on
the river Chenab. • Statement 1 is incorrect: Shat al Arab is a river
• Statement 3 is incorrect: The Indus River in Iraq that is formed due to the confluence of
System comprises the five rivers flowing in the Tigris and the Euphrates River. It drains
India and Pakistan. They are Indus, Jhelum, into the Persian Gulf and is not associated with
Chenab, Ravi, Beas, and Sutlej arranged in the the Red Sea.
North to South direction. Hence, Indus is the • Statement 2 is incorrect: Sharm El Sheikh is a
northernmost while Sutlej is the town of Egypt and is located at the
southernmost river of the system. southernmost tip of the Sinai Peninsula along
the coast of the Red Sea. It is not associated
Source: with the Persian Gulf.
https://www.thehindu.com/news/national/talks-

DAY 34_IDMP FULL LENGTH TEST 5 37


Contact us : info@onlyias.com

OnlyIAS Nothing Else Visit : dpp.onlyias.in


Contact : +91-7007 931 912

• Statement 3 is correct: Port Said is a part of in different geographical locations and hence
Egypt. It is located in the Northern Part of are named differently. Apatani is associated
Egypt along the Mediterranean Sea. with the Eastern Himalayas. Zabo is linked to
• Statement 4 is correct: Muscat is located along the Northeastern Hill ranges. Dighis are the
the northeastern part of Oman. It is the capital water harvesting systems found in the Indo-
of Oman. It lies on the Arabian Sea and is a part gangetic plains. Paar is associated with the
of the Gulf of Oman. It is located remarkably Thar Desert region. Jampois are related to the
close to the strategic strait of Hormuz. Brahmaputra Valley.

Source: Atlas Source: https://www.cseindia.org/traditional-water-


harvesting-systems-683
Q.84) Ans: D
Q.86) Ans: D
Exp:
Exp:
• Statement 1 is incorrect: The idea of BRICS was
first mooted by Goldmann Sachs in 2001 and • Statement 1 is incorrect: National Bank for
the first official summit was organized in 2009 Financing Infrastructure and Development
in Russia. In the 2010 summit South Africa was (NABFID) is a Developmental Financial
added to the BRICS network which was initially Institution that seeks to provide long term
formed by Brazil, Russia, India, and China. At finance to various sectors for infrastructure
the Fortaleza Summit in 2014, the creation of development. It is set up as a corporate body
the New Development Bank was done to with initial central government shareholding of
strengthen the financial and the economic 100%. It would provide investment to the
cooperation of the group. Hence, it was infrastructure projects under the National
formed after the addition of South Africa in Infrastructure Pipeline. The chairperson of the
2010. The headquarters of the NDB is located NABFID would be appointed by the Central
at Shanghai, China and is formed with equal Government in consultation with the RBI.
contribution from each partner. Commercial banks are set up under the
• Statement 2 is incorrect: Gulf Cooperation Companies Act whereas NABFID is set up
Council (GCC) was established in 1981 among under the specialized acts enacted for this
Bahrain, Kuwait, Oman, Qatar, Saudi Arabia, purpose.
and UAE as a political, economic, regional • Statement 2 is incorrect: NABFID can raise
organization of the above countries based on money in the form of loans and otherwise. It
their common objectives, interests, and the can raise finance both in Indian and foreign
Islamic beliefs. Iran is a country bordering the currencies. It can also issue various financial
Persian Gulf. However, it is not a part of the instruments like the Bonds, debentures, etc.
GCC. for the purpose of its financing.
• Statement 3 is incorrect: The aim of the
Source: NABFID is not just to lend but also to invest or
https://pib.gov.in/Pressreleaseshare.aspx?PRID=1704 attract investments in various sectors for the
262 ; https://infobrics.org/ development of infrastructure. It can finance
the projects situated not just within India but
https://www.gcc-sg.org/en-us/Pages/default.aspx also projects that are partly located in India.

Q.85) Ans: B Source:


https://indianexpress.com/article/business/economy/
Exp: dfi-to-raise-3-lakh-cr-for-infra-spend-20k-cr-to-be-
infused-7231720/
• Option B is correct: Apatani, Zabo, Dighis,
Paar, Jampois are the various traditional Q.87) Ans: B
water management systems. They are found
DAY 34_IDMP FULL LENGTH TEST 5 38
Contact us : info@onlyias.com

OnlyIAS Nothing Else Visit : dpp.onlyias.in


Contact : +91-7007 931 912

Exp: and hence its yield is increased and vice versa.


Greater yield of Bond compared to the stocks
• Statement 1 is incorrect: National Payments of a Company shifts the investors to the Bond
Corporation of India or the NPCI is the market and hence the prices of stocks have to
umbrella entity for settling all retail payments be lowered to attract greater investors.
and settlements systems of India. It has been • Statement 2 is correct: Greater Bond yield of
incorporated under the Payments and the USA government bonds encourages the
Settlements Act, 2007. It is a not-for-profit investors to invest in them rather than other
company established under the Companies countries with lower returns on investments.
Act, 1956 (now 2013). It was an initiative of the Hence, any change in the bond yield of the USA
RBI and the Indian Banks’ Association (IBA). impacts investment in other countries.
Several payment related platforms are • Statement 3 is incorrect: Capital Gains tax is
operated by the NPCI which includes the Rupay charged only when there occurs a sale or
cards, Unified Payment Interface, Bharat Bill purchase of a capital asset. Hence, stocks or
Payments, FASTag, National Automated shares that are continuously incurring profits
Clearing House, National Financial Switch, would not be subjected to the Capital Gains
Aadhar Enabled Payments, etc. It has not Tax unless they are sold.
incorporated as the Critical Payment
Infrastructure Company as such. However, it Source:
was the Watal Committee that suggested the https://economictimes.indiatimes.com/markets/bond
classification of the NPCI as the Critical s/explainer-what-rising-bond-yields-mean-for-
Payment Infrastructure Company of India. markets/articleshow/81602095.cms?from=mdr
• Statement 2 is correct: Cyber Swachhta
Kendra is an initiative of the Ministry of Q.89) Ans: C
Electronics and Information Technology to
create a secure cyberspace by detecting Exp:
Botnet infections. Hence, it is a Botnet
Cleaning and Malware Analysis Centre. • Statement 1 is incorrect: Recently, NITI Ayog
and Rocky Mountain Institute of India came
Source: https://www.cyberswachhtakendra.gov.in/ up with the report titled “Mobilising Electric
Vehicle Financing in India”. The adoption of
https://www.npci.org.in/who-we-are/about-us Electric Vehicles would play an instrumental
role in the protection of the Environment. In
order to boost the demand for the Electric
Vehicles in India, there was a reduction in the
Q.88) Ans: C GST rate from 12 to 5%.
• Statement 2 is incorrect: Vehicle Scrapping
Exp: Policy was announced by the Ministry of Road
Transport and Highways. As per the provisions
• Statement 1 is incorrect: Bond is a fixed of these, Commercial vehicles need to be de-
income instrument that is used by a borrower registered after 15 years. The same time limit
to raise a loan. It can be issued by a corporation for Private Vehicles is 20 years while that of
or the government. It has a maturity date on Government Vehicles is 15 years. It also
which the issuer pays the entire face value of proposes incentives for the scrapping of the
the bond to the investor. The coupon rate old and unfit vehicles and setting up highly
refers to the rate of interest the issuer pays to specialized registered vehicle scrapping
the investor. Generally, the term bond is used facilities across the country.
to refer to the government bonds. Bond yield
refers to the return offered on the investment Source:
in a bond. It is inversely proportional to the https://www.timesnownews.com/auto/features/articl
Bond prices. When the demand for a bond is
higher, its price falls in the secondary market

DAY 34_IDMP FULL LENGTH TEST 5 39


Contact us : info@onlyias.com

OnlyIAS Nothing Else Visit : dpp.onlyias.in


Contact : +91-7007 931 912

e/government-to-offer-subsidy-to-62000-electric- Q.91) Ans: B


cars-and-buses/719482
Exp:
https://www.thehindu.com/business/Industry/scrapp
age-policy-will-boost-auto- • Statement 1 is incorrect: The Global Hunger
sector/article34112091.ece Index is released by the Concern Worldwide
and Welthungerhilfe. It consists of four
Q.90) Ans: D indicators and three dimensions. In its 2020
edition, India has been ranked at 94th spot
Exp: lower than even Bangladesh and Pakistan. As
per the report,
• Option A is incorrect: The National Food o 14% of Indians lack enough calorie
Security Act, 2013 aims to provide subsidized food,
food grains to the eligible households. There is o around 35% of children are stunted,
a provision of 5 kg per month per person of o 17.3% of Indian children under the age
foodgrains for the priority households along of 5 are wasted, and
with the 35 kg foodgrains per month for the o there exists 3.7% of under-five
households under the Antyodaya Anna Yojana. mortality rates.
As per the provisions of the Act, the Central • Statement 2 is correct: World Happiness
Issue price needs to be revised after every report is released by the UN Sustainable
three years. This has not been done yet and is Development Solutions Network. The theme
one of the reasons for the rising food subsidy of 2021 was the Effects of COVID 19 on
bill of the government. happiness. India was ranked at 139th spot out
• Option B is incorrect: The Act outlines the of 149 countries. Bangladesh was ranked at
responsibilities of not only the Central but also 101st and showed better performance than
the state governments. The identification and India.
the issue of license to the Fair Price shops is to
be done by the State Government. Source:
• Option C is incorrect: Shanta Kumar https://www.thehindu.com/news/national/agricultur
Committee in 2015 had called for the revision e-mos-questions-global-hunger-index-reports-
of the list of beneficiaries to reduce the methodology/article34107191.ece
number of beneficiaries covered. Increasing
population has put tremendous pressure on https://www.livemint.com/news/india/india-ranks-
the government’s subsidy bill and hence there 139-in-world-happiness-report-here-s-list-of-20-
is a need to rationalize the scheme. happiest-countries-11616202779157.html
• Option D is correct: The rise of the food
subsidy bill is attributed to the difference in the Q.92) Ans: D
prices at which the purchase of grains at MSP
is done by the FCI (Food Corporation of India) Exp:
followed by its storage and distribution and
that of the Central Issue Price (CIP), price at • Statement 1 is incorrect: Large Hadron
which these grains are provided for Public Collider is the most powerful particle
Distribution System. The CIP is much lesser accelerator located along the French and Swiss
than the cost incurred in its procurement, border. It contains several powerful
transportation, and distribution leading to superconducting magnets to accelerate the
higher food subsidy bill. matter as it passes through it. It is housed at
the complex of European Organization for
Source: Nuclear Research or CERN. It announced the
https://www.thehindu.com/news/national/census- discovery of the Higgs boson in 2012 and
data-may-decide-food-subsidy/article34055061.ece mainly deals with the study of the fundamental
mrunal or elementary particles. Black Holes are

DAY 34_IDMP FULL LENGTH TEST 5 40


Contact us : info@onlyias.com

OnlyIAS Nothing Else Visit : dpp.onlyias.in


Contact : +91-7007 931 912

studied by the Event Horizon Telescope and Source:


not by the LHC. https://www.thehindu.com/news/national/andhra-
• Statement 2 is incorrect: The particles at the pradesh/kendra-sahitya-akademi-award-for-velcheru-
LHC are made to accelerate at a speed narayana-rao/article33945737.ece
approximately equal to the speed of light. The
particles are accelerated at such a high speed https://www.icar.org.in/content/indian-council-
and to collide with each other. The change in agricultural-research-won-king-bhumibol-world-soil-
their properties or the nature of their day-award-2020
properties are hence studied.
• Statement 3 is incorrect: CERN is the one of https://pib.gov.in/PressReleasePage.aspx?PRID=1704
the largest centers of research that studies the 434
nature of the fundamental particles. It was
founded in 1954 and is located near Geneva. https://www.thehindu.com/news/national/other-
India is an associate member state of CERN, states/pm-modi-slammed-for-calling-17th-century-
which has 23 member states. Japan, USA, ahom-general-a-freedom-
Russia, EU, and UNESCO have been accorded fighter/article34066126.ece
the observer status.
Q.94) Ans: A
Source:
https://home.cern/news/news/physics/intriguing- Exp:
new-result-lhcb-experiment-cern
• Statement 1 is correct: Coronal Mass Ejections
https://home.cern/about/who-we-are/our-mission (CMEs) is a huge cloud of solar plasma
equipped with magnetic field lines that are
Q.93) Ans: D blown away from the surface of the Sun
leading to strong solar flames. It is mostly
Exp: composed of particles like protons and
electrons that are radiated with strong
• Statement 1 is correct: Professor Velcheru magnetic fields. This adversely impacts the
Narayana Rao was conferred with the Sahitya space weather, causes power outages, and the
Akademi Fellowship for his contribution to the failure of the satellites, etc.
field of Telugu literature. He was born in • Statement 2 is incorrect: ARKTIKA-M is a
Koppaka near Eluru, Andhra Pradesh. His Russia’s monitoring satellite to monitor the
important works include: Girls for sale: environment of the Arctic region. It would not
kanyasulkam, etc. only look into the climate of the Arctic but
• Statement 2 is correct: Indian Council for would also lead to huge opportunities for the
Agricultural Research was awarded with the exploration of the natural resources in the
King Bhumibol World Soil Day Award, 2020. region. It does not cover the two polar regions
King Bhumibol Adulyade was the king of the rather only covers the Arctic region.
Chakri dynasty of Thailand. He was the longest
serving monarch of Thailand. Source:
• Statement 3 is correct: Ayya Vaikunda https://pib.gov.in/PressReleasePage.aspx?PRID=1708
Swamikal was a great social reformer 735
belonging to 19th Century Kerala. He fought
against untouchability and idol worship and https://www.thehindu.com/sci-tech/science/russia-
founded the Samathwa Samajam. launches-satellite-to-monitor-climate-in-
• Statement 4 is correct: Lachit Borphukan was arctic/article33959352.ece
the king of the Ahom dynasty of Assam who
defeated the Mughals in 1671 at the Battle of Q.95) Ans: B
Saraighat in the Brahmaputra valley.
Exp:

DAY 34_IDMP FULL LENGTH TEST 5 41


Contact us : info@onlyias.com

OnlyIAS Nothing Else Visit : dpp.onlyias.in


Contact : +91-7007 931 912

• Statement 1 is correct: A supercomputer is a ▪ Cultivation of prioritized


fast-acting computer that exhibits faster medicinal plants on farmer’s
performance compared to other ordinary land.
computers. They work very quickly owing to ▪ Establishment of nurseries
the parallel processing of information as with backward linkages for
against the ordinary computers that perform raising and supply of quality
one task at a time and work in a serial manner. planting material.
• Statement 2 is incorrect: The speed of ▪ Post-harvest management
microprocessors is measured in FLOPS or the with forward linkages.
floating operations per seconds and it ▪ Primary processing,
indicates the performance speed of the marketing infrastructure etc.
computers. Bits or bytes refer to the memory o Under this scheme, subsidy at the rate
or storage information of the computer. of 30%, 50% and 75% of the cost of its
• Statement 3 is incorrect: Japan’s Fugaku has cultivation is provided for cultivation
become the world’s fastest supercomputer of 140 prioritized medicinal plants on
with a speed of around 415 petaflop. India’s farmer’s land.
fastest supercomputer, PARAM Siddhi has the
speed of 210 petaflops which is around half of NEWS:
Fugaku. India started the National https://pib.gov.in/PressReleasePage.aspx?PRID=170
Supercomputing mission in 2015 to aid 4851
research in the area. PARAM Siddhi, PARAM
Shivay, PARAM Shakti, PARAM Brahma are Q.97) Ans: A
the supercomputers developed under this Exp:
mission. • Statement 1 is correct:
o The raw silk production in the country
Source: https://www.thehindu.com/sci- has shown an increasing trend during
tech/technology/worlds-most-powerful- the last five years.
supercomputer-is-ready-for-use/article34043150.ece o India has the unique distinction of
being the only country producing all
https://nsmindia.in/ the four kinds of silk namely,
Mulberry, Eri, Muga, Tasar (Tropical
and Temperate both). Mulberry silk is
the most popular variety, which
Q.96) Ans: B
contributes around 79% of the
country’s silk production.
Exp:
• Options 1,3 and 4 are correct:
o The Ministry of AYUSH is presently
implementing the Centrally
Sponsored Scheme of National
AYUSH Mission (NAM). Under the
‘Medicinal Plants’ component of the
NAM scheme, The Govt. is supporting
market driven cultivation of
prioritized medicinal plants in
identified clusters/zones within
selected districts of States in a mission • Statement 2 is incorrect:
mode throughout the country. As per o The Govt. of India through Central Silk
the scheme guidelines, the support is Board has been implementing a
provided for: Central Sector Scheme "Silk Samagra"
an Integrated Scheme for
Development of Silk Industry (ISDSI)

DAY 34_IDMP FULL LENGTH TEST 5 42


Contact us : info@onlyias.com

OnlyIAS Nothing Else Visit : dpp.onlyias.in


Contact : +91-7007 931 912

during the years 2017-18 to 2020-21 Conference in 1960, by Iran, Iraq,


for the overall development of Silk Kuwait, Saudi Arabia, and Venezuela.
industry in the Country with an aim & It is headquartered in Vienna.
objective to scale up production by o It aims to manage the supply of oil in
improving the quality and productivity an effort to set the price of oil in the
and to empower downtrodden, poor world market, in order to avoid
& backward families through various fluctuations that might affect the
activities of sericulture in the country economies of both producing and
o Central Silk Board (CSB) is a Statutory purchasing countries.
body established in 1948 by an Act of o The Organization of the Petroleum
Parliament. It is working under the Exporting Countries Plus (OPEC+) is a
administrative control of the Ministry loosely affiliated entity consisting of
of Textiles, Government of India. the 13 OPEC members and 10 of the
world's major non-OPEC oil-exporting
NEWS: nations. OPEC+ aims to regulate the
https://pib.gov.in/PressReleasePage.aspx?PRID=170 supply of oil in order to set the price on
6034 the world market.

Q.98) Ans: B o OPEC plus countries include


Exp: Azerbaijan, Bahrain, Brunei,
• Option B is correct: Kazakhstan, Malaysia, Mexico, Oman,
• Dholavira is an archaeological site at Russia, South Sudan and Sudan.
Khadirbet, Rann of Kutch, Gujrat. Also known
locally as Kotada timba, the site contains ruins
of an ancient Indus Valley
Civilization/Harappan city. Dholavira’s
location is on the Tropic of Cancer. It is one of
the five largest Harappan sites and most
prominent archaeological sites in India
belonging to the Indus Valley Civilization.
o Dholavira: A Harappan City’ has been
submitted for nomination as a World
Heritage Site in 2019-2020.
Additionally Nomination dossiers of
‘Santiniketan, India’ and ‘Sacred NEWS: ‘OPEC+ move to hit recovery’ - The Hindu
Ensemble of Hoysalas’ have been SOURCE: OPEC : Member Countries
submitted to UNESCO for the year
2021-22 cycle.
Q.100) Ans: B
NEWS: Declaration Of World Heritage Sites By Exp:
UNESCO (pib.gov.in) Option B is correct:
SOURCE: Ruins on the Tropic of Cancer • India has been placed among the top 10 most
affected countries by climate change, in the
Global Climate Risk Index 2021 published by
Q.99) Ans: B the Bonn-based environmental think tank
Exp: German watch. India ranked as the seventh
• Options 1,2 and 5 are correct: worst-hit country in terms of climate change in
o The Organization of the Petroleum 2019.
Exporting Countries (OPEC) is a • The Global Climate Risk Index 2021 is the 16th
permanent, intergovernmental edition of the annual report and has taken into
organization, created at the Baghdad account the data available for 2019 and from

DAY 34_IDMP FULL LENGTH TEST 5 43


Contact us : info@onlyias.com

OnlyIAS Nothing Else Visit : dpp.onlyias.in


Contact : +91-7007 931 912

2000 to 2019. The index analyses and ranks to


what extent countries and regions have been
affected by impacts of climate-related extreme
weather events (storms, floods, heatwaves
etc.)

DAY 34_IDMP FULL LENGTH TEST 5 44

You might also like